SóProvas



Prova Aeronáutica - 2021 - EEAR - Controle de Tráfego Aéreo


ID
5338000
Banca
Aeronáutica
Órgão
EEAR
Ano
2021
Provas
Disciplina
Português
Assuntos

Em qual altenativa o pronome oblíquo átono está corretamente colocado?

Alternativas
Comentários
  • Não lhe dariam o cargo se não fosse competente para exercer tal função.

    Em frases que demonstram um sentido de negação, ocorrerá a próclise (que é a colocação do pronome ANTES do verbo).

    GABARITO: Letra C

  • a) Me indicaram ao cargo, mas não sou o melhor candidato

    Errada, não podemos começar frase com pronomes oblíquos átonos

    b) Nós havíamos indicado-lhe vários candidatos merecedores do cargo

    Errada, o correto creio que seria ter um s no final do lhe pra manter a ordem da frase ( Caso não seja isso, pode corrigir !)

    c) Não lhe dariam o cargo se não fosse competente para exercer tal função.

    Correta, palavras negativas atraem pronomes oblíquos átonos

    d) Em tratando-se de eficiência, ele deu provas suficientes para merecer o cargo

    Errada, Preposição sempre atrai pronome oblíquo átono; logo o correto seria " Em se tratando..."

    BIZU DA PRÓCLISE

    P reposição + Gerúndio

    A dvérbio

    I ndefinido

    R elativo

    I nterrogativo

    C onjunção

    O ptativo

    @maurofilho87

  • Mauro a alternativa B sua está errada, pois não se pode colocar pronome depois do verbo principal no particípio, essa é a regra que derrubar essa alternativa. O resto está certo :)

    A

    Me indicaram ao cargo, mas não sou o melhor candidato. Pronome não pode iniciar frases em nenhuma hipótese.

    B

    Nós havíamos indicado-lhe vários candidatos merecedores do cargo. Estando o verbo principal no particípio, o pronome oblíquo átono não poderá vir depois dele.

    C

    Não lhe dariam o cargo se não fosse competente para exercer tal função. O GABARITO, pois a partícula atrativa NÃO pede PROCLISE

    D

    Em tratando-se de eficiência, ele deu provas suficientes para merecer o cargo. Quando se tem a expressão "em + gerúndio" pede próclise no gerúndio, logo o certo seria "Em se tratando"

  • PRONOME OBLÍQUO ÁTONO:ME,TE,SE,NOS,VOS,O(S),A(S),LHE(S)

    ÊNCLISE-OBRIGATÓRIA QUANDO O PRONOME VEM LIGADO A UM VERBO QUE INICIA A ORAÇÃO

    AJUDOU A TODOS OS ALUNOS

    AJUDAR=VERBO

    A TODOS OS ALUNOS=OBJETO INDIRETO

    AJUDOU-LHES

    LHES=PRONOME OBLÍQUO ÁTONO

    PRÓCLISE-SERÁ OBRIGATÓRIA SEMPRE QUE HOUVER FATORES PRÓCLITICOS

    PREPOSIÇÃO EM+GERÚNDIO

    ADVÉRBIO

    INTERROGATIVO

    RELATIVO

    INDEFINIDO

    CONJUNÇÕES

    ORAÇÕES OPTATIVAS

    VERBOS TERMINADOS EM M/~=ACRESCENTAM N

    ESTÃO ESTUDANDO AQUELE ASSUNTO

    AQUELE ASSUNTO-OBJETO DIRETO

    ESTÃO-NO ESTUDANDO

    VERBOS TERMINADOS EM R/S/Z= ACRESCENTAM L

    VOU AJUDAR MEUS AMIGOS

    MEUS AMIGOS=OBJETO DIRETO

    VOU AJUDÁ-LOS

    GAB:C

  • Quando o verbo principal está no particípio, o pronome átono fica me relação ao verbo auxiliar

  • O "não" puxa o pronome, dessa forma ocorrerá próclise.

    GAB LETRA C

  • NÃO SE INICIA ORAÇÃO COM POA (PRONOME OBLÍQUO ÁTONO)

  • Palavra negativa atrai forte a próclise

    Mas no caso da D tem algo muito incomum de ocorrer e vale lembrar

    Nos casos que temos Em+gerúndio utiliza-se a próclise de forma obrigatória, somente nesse tipo de estrutura

    GAB C

    APMBB

  • Palavra negativa trás a próclise.


ID
5338003
Banca
Aeronáutica
Órgão
EEAR
Ano
2021
Provas
Disciplina
Português
Assuntos

Assinale a alternativa cuja classificação da figura de linguagem está incorreta.

Alternativas
Comentários
  • Juliano agora é funcionário da limpeza pública. Está muito feliz com o novo emprego na Prefeitura de Lindópolis. (metáfora) INCORRETA.

    Configura-se zeugma pela omissão do Juliano ou Elipse pela omissão do pronome "ELE"

    Corrijam-me se houver erro.

  • Antítese: Ideias contrarias

    letra A: Na frase: Eis que era cristalina e vibrante, / Eis-me turvo e triste. possui uma ideia de contrariedade entre (vibrante/triste)

    Metonímia: Uso de palavras fora do seu contexto semântico normal

    letra B: Na frase: A empregada enxuga as porcelanas cuidadosamente, pois não quer quebrar uma peça sequer.

    possui uma palavra fora do contexto (uma peça sequer) pois acredito eu que se uma porcelana por esta nas mãos de uma pessoa caso ela cair, ela irá se quebrar toda. kkkkk

    Prosopopeia ou personificação: Características humanas

    letra C: Na frase: Fora da casa, só silêncio, um grande silêncio, e o vento ficou esperando, amarrado na soleira da porta.

    possui uma palavra com características humanas ( amarrado)

    Metáfora: Comparação

    letra D: Na frase: Juliano agora é funcionário da limpeza pública. Está muito feliz com o novo emprego na Prefeitura de Lindópolis.  a frase não esta fazendo comparação, apenas afirmando que ele esta feliz com o novo emprego

    ESPERO TER AJUDADO.

    RESPONDI COM OS MEUS CONHECIMENTOS, ESTOU SUJEITA AO ERRO, ENTÃO CORRIJA-ME SE ESTIVER ERRADO.

  • A alternativa "d" está incorreta porque se trata da elipse do sujeito que é o "Juliano".


ID
5338006
Banca
Aeronáutica
Órgão
EEAR
Ano
2021
Provas
Disciplina
Português
Assuntos

Assinale a alternativa que não está de acordo com a norma culta quanto à regência dos nomes em destaque.

Alternativas
Comentários
  • admiração por

    • Se trata de uma questão re regência nominal e infelizmente você tem que saber alguns que exigem a preposição certa de cabeça e lembrando que ele pede a errada

    a) São poucas as funções a que esta jovem trabalhadora está apta

    Correta, pois a palavra apta exige as preposições a ou para quem está apta está apta a algo ou está apta para algo

    b) O pai recriminava os hábitos culturais que a filha tinha admiração.

    Errada, quem tem admiração tem admiração por algo ou por alguma coisa, logo a preposição correta seria Por

    c) As críticas a que o chefe era sensível agora não o incomodam mais.

    Correta, quem é sensível é sensível a algo ou a alguma coisa

    d) Os profissionais a quem ele tem desprezo fizeram com que ele perdesse o emprego.

    Correta, a palavra desprezo exige as preposições a, de , por

    @maurofilho87

  • E como seria a segunda frase se ela estivesse correta?

  • Ela está apta a funções

    A filha tinha admiração aos hábitos

    Ele era sensível as críticas

    Ele despreza os profissionais


ID
5338009
Banca
Aeronáutica
Órgão
EEAR
Ano
2021
Provas
Disciplina
Português
Assuntos

Quanto ao uso ou não do acento grave indicador de crase, assinale a alternativa que preenche, correta e respectivamente, as lacunas do texto abaixo.

“Minha mãe, pessoa mais intransigente da casa, um dia acordou aberta _____ novas experiências. Deixou de lado seu ponto de vista contrário _____ aquisição de animais domésticos e achou que valeria _____ pena adotar um cão. Porém, quanto _____ passarinhos, jamais; queria-os livres para voarem rumo ______ liberdade.”

Alternativas
Comentários
  • CRASE PROIBIDA:

    -   A no singular + palavra no plural, CRASE, passa mal.

    Ex.: A inserção do sinal indicativo de crase em “a interpretações” (ℓ.7) ocasionaria erro gramatical no texto.

    A bioeconomia está ligada a melhorias de nosso desenvolvimento e buscas por novas tecnologias

     -  Ex.: Habitam (VTD) a casa / Terra (SEM crase)

    Crase é proibida ANTES DE NUMERAL. (exceto para horas) Ex.:  Chegaram A um metro de altura

    -Locução Adverbial de Instrumento.    Ex: Jessé cortar cabelo a tesoura;

    Locução Adverbial de Meio.    Ex: Viajaram num barco a vela;

    -Palavras repetidas.   Ex: passo a passo; gota a gota; uma a uma;

    -Antes de QUEM; CUJA; (exceções à QUE)

    -Pronomes de Tratamento:    Vossa Excelência, Vossa Senhoria

    -Pron. Pessoal do caso Reto:       EU, TU, ELE, NÓS, VÓS, ELES

    -Pron. Demonstrativo:        ESTA, ESSA, AQUELA

    -Pron. Indefinido:     ALGUM, NENHUM, TODA, ALGUÉM, NADA, QUALQUER, POUCO, CERTO, VÁRIOS, UMA

    ANTES DE VERBO. Ex: destinada a receber

    Ex.: "De uma geração à outra."

        "De uma geração à outra (geração)."

    Eu não vou desistir e nem você!!!

    • Primeiro quadrinho não temos crase pois diante de plural crase não entra

    OBS: B e C cai fora

    • Segundo quadrinho vai crase pois quem é contrário é contrário a algo ou a alguma coisa e aquisição é palavra feminina logo a ( preposição) + a (artigo) = à

    OBS: Matei a questão rsrs

    • No terceiro quadrinho não vai crase se pergunte acho que valeria o que? Como o termo não pede preposição, não temos crase

    • No quarto quadrinho não vai crase pois diante de plural crase não entra

    • No quinto quadrinho vai crase pois quem vai rumo vai rumo a algo ou a alguma coisa , perceba que exige uma preposição e a palavra liberdade é feminina com isso a ( preposição) + a (artigo) = à

    BIZU DA CRASE

    Diante de pronome, crase passa fome.

    Diante de Masculino, crase é pepino.

    Diante de ação, crase é marcação.

    Palavras repetidas: Crases proibidas.

    “A” + “Aquele” = Crase nele!

    Vou a, volto da, então crase há!

    Vou a, volto de, crase para quê?

    Diante de cardinal, crase faz mal.

    Quando for hora, crase sem demora.

    Palavra determinada, crase liberada.

    Sendo à moda de, crase vai vencer.

    Adverbial, feminina e locução! Manda crase, meu irmão!

    @maurofilho87

  • mauro luiz gostei do bizuuuuuuuuuuuu


ID
5338012
Banca
Aeronáutica
Órgão
EEAR
Ano
2021
Provas
Disciplina
Português
Assuntos

Relacione as colunas e, em seguida, assinale a alternativa com a sequência correta.

1 – objeto direto ( ) Estava confiante na vitória.
2 – objeto indireto ( ) Há grandes festejos naquele bairro.
3 – complemento nominal ( ) Cedeu aos caprichos infantis.
( ) Não me convidou para o lanche.
( ) Peço-lhe paciência com os jovens.

Alternativas
Comentários
  • (3) Estava confiante na vitória.

    Complemento Nominal, pois confiante é um substantivo abstrato , logo de temos substantivo abstrato + preposição = C.N

    OBS: Cai fora B e C

    (1) Há grandes festejos naquele bairro.

    O.D, se pergunto o que há naquele bairro? Automaticamente você responderá Grandes Festejos , perceba que não exigiu preposição portanto se trata de O.D

    OBS: Matei a questão

    (2) Cedeu aos caprichos infantis.

    O.I, perceba que a palavra Ceder exigi preposição ; Portanto O.I

    (1) Não me convidou para o lanche.

    O.D, perceba que Quem convida convida alguém logo V.T.D o que vai me dar um O.D

     (2) Peço-lhe paciência com os jovens.

    O.I, Pedir é um V.T.D.I porque perceba quem pede pede algo a alguém , pede o que? Paciência e ele pede a quem a LHE ( A ti) perceba que exige preposição logo o lhe é O.I e paciência é O.D

    @maurofilho87


ID
5338015
Banca
Aeronáutica
Órgão
EEAR
Ano
2021
Provas
Disciplina
Português
Assuntos

Em relação ao emprego do pronome relativo, marque C para certo e E para errado. Em seguida, assinale a alternativa com a sequência correta.

( ) Quero apresentar-lhe a filha cuja a mãe dela foi eleita a Mãe do Ano.
( ) Posso saber o motivo que você desistiu de concorrer àquele prêmio tão sonhado?
( ) O futebol é o esporte pelo qual os homens brasileiros mais se interessam.
( ) As dificuldades por que passamos servem para nos tornar mais fortes na caminhada.

Alternativas
Comentários
  • 1- E- Depois de cuja não pode ter artigo.

    2- EEsse que É conjunção integrante, substitui por diSSO.

    3- C- Quem se interessa se interessa por algo, então pelo qual está certinho. por + o

    4- C- Substitui por qual se fizer sentido está certo kkkk

    obs: corrijam-me se necessário.

  • o "que" é pronome demonstrativo = pelo qual
  • Cujo geralmente fica no meio de dois substantivos, ali está com artigo do lado

    E ''Aquele'' está grafado incorretamente, eliminando já chega na resposta

    GAB C

    APMBB

  • Fala, galera!

    Correção em vídeo: https://www.youtube.com/watch?v=hG2OxrMf85k QUESTÃO 20


ID
5338018
Banca
Aeronáutica
Órgão
EEAR
Ano
2021
Provas
Disciplina
Português
Assuntos

Marque a alternativa em que há erro na classificação do termo em destaque.

Alternativas
Comentários
  • a) “Para um homem se ver a si mesmo, são necessárias três coisas: olhosespelho e luz.” (Pe Antônio Vieira) - aposto

    Correta, se trata de um aposto enumerativo

    b) "Ofendi-vos, meu Deus, é bem verdade” (Gregório de Matos) - vocativo

    Correta, perceba que tá chamando por Deus logo se trata de um vocativo

    c) Sete anos de pastor Jacó servia / Labão, pai de Raquel, serrana bela” (Luís de Camões) - aposto

    Correta, se trata de um Aposto especificativo pois está especificando quem é o Labão no caso o Pai de Raquel

    d) “Este lugar delicioso, e triste, / Cansada de viver, tinha escolhido / Para morrer a mísera Lindoia.” (Basílio da Gama) - aposto

    Errada

    @maurofilho87

  • Seria a alternativa D) um adjunto adnominal?

  • @Kleber Henrique creio que sim. pois adjunto adnominal acompanha substantivo e não é preposicionado. se tivesse preposição seria complemento nominal. tomara que tenha te esclarecido. corrijam-me se eu estiver errado ✌
  • Alternativa D é predicativo do sujeito

  • Este lugar delicioso, e triste - adjunto adverbial de lugar

  • Creio que seja predicativo do objeto, pois quando passo a frase para a ordem direta fica:

    A mísera Lindoia, cansada de viver, tinha escolhido este lugar delicioso, e triste para morrer.

    este lugar é objeto direto do verbo escolher, onde "delicioso, e triste" caracteriza o objeto.

  • D é Adjunto Adnominal. Quando li o comentário do Alexandre fiquei um pouco na dúvida, mas lembrei do macete de trocar por pronome átono. Se o termo desaparece junto com o objeto, é acessório (adjunto adnominal), se permanece na oração é atributo (predicativo do objeto).

    Assim: A mísera Lindoia, cansada de viver, tinha escolhido este lugar delicioso, e triste para morrer.

    A mísera Lindoia, cansada de viver, tinha o escolhido para morrer.

    Portanto, é ADJUNTO ADNOMINAL.


ID
5338021
Banca
Aeronáutica
Órgão
EEAR
Ano
2021
Provas
Disciplina
Português
Assuntos

Assinale a alternativa que completa, correta e respectivamente, as lacunas seguintes.

(1) Temos ____ conhecidos na cidade.
(2) São ____ risonhos com quem simpatizam.
(3) Remeti ____ a nota fiscal.
(4) Os adultos chegaram ____, e as crianças vieram ____ com a professora.

Alternativas
Comentários
    • Primeiro quadrinho substituirmos por Bastantes pois quando posso trocar por Muitos e a frase mantém sentido completamos com Bastantes, caso der ideia de Muito completamos com Bastante

    OBS: A e B cai fora

    • Segundo quadrinho da ideia de muito, logo completamos com Bastante

    • Sempre que depois das palavras Incluso, Anexo e Leso vier um artigo ela irá variar, portanto Anexa seria o correto, lembrando que Em Anexo é invariável

    OBS: Matou a questão

    • O quarto quadrinho completamos com Juntos no plural pois remete aos Adultos , portanto tem que ficar no plural

    • No último quadrinho completamos com Junto pois vai concordar com as crianças, não pode ser Juntas

    @maurofilho87

  • Bastante

    1ªcaso: Quando tiver o valor de advérbio de intensidade ( muito ou suficientemente) será INVARIÁVEL

    Estes exercícios são bastante fáceis.

    2ªcaso: Como pronome indefinido, acompanhado o substantivo ( equivalente a muitos, suficientes) é VARIÁVEL

    Duas malas não eram bastantes para minha esposa.

  • ANEXO: INCLUSO E APENSO : São termos variáveis que devem concordar com o substantivo.

    EX: Segue ANEXAS as IMAGENS descritas

    Seguem INCLUSAS as provas

    Segue APENSOS os DOCUMENTOS.


ID
5338024
Banca
Aeronáutica
Órgão
EEAR
Ano
2021
Provas
Disciplina
Português
Assuntos

Há sujeito simples em qual alternativa?

Alternativas
Comentários
  • Gabarito D

    Não há sujeito quando verbo Haver tem sentido de Existir, como ocorre nas alternativas a, b e c.

    Perceba que, na alternativa D, o verbo principal não é o HAVER, mas sim o verbo PARTIR.

    Minha esperança de felicidade ( sujeito ) havia partido ( tempo composto) no lombo do cavalo baio( adjunto adverbial)

  • Verbo Haver no sentindo Existir = Sem Sujeito

    @maurofilho87

  • a) “Mais ao longe, numa volta da estrada, uma esperança havia.”

    verbo haver no sentido de existir = oração sem sujeito.

    b) “Nenhuma pedra poderia haver no caminho da felicidade.”

    verbo principal haver no sentido de existir = oração sem sujeito

    c) “Nunca houve cometa igual, assim tão terrível, desdenhoso e belo.”

    verbo haver no sentido de existir = oração sem sujeito.

    d) “No lombo do cavalo baio, havia partido minha esperança de felicidade."

    verbo principal é PARTIR, logo o sujeito da oração é MINHA ESPERANÇA


ID
5338027
Banca
Aeronáutica
Órgão
EEAR
Ano
2021
Provas
Disciplina
Português
Assuntos

Em qual alternativa as três palavras estão corretamente acentuadas?

Alternativas
Comentários
  • a) Não são acentuados os hiatos seguidos por nh ou quando, na mesma sílaba, acompanharem outra letra que não seja s , portanto , Juiz e moinho não são acentuadas

    b) Gabarito correto, saímos ( hiato ), Lençóis e pastéis (oxítonas- ditongo aberto )

    c) amendoim não é acentuada pois é uma oxítona terminada em im

    d) Heroico e chapeuzinho perderam o acento após o novo acordo ortográfico ( ditongos éi, ói e éu perdem o acento quando vierem na penúltima sílaba, ou seja, quando forem paroxítonos )

  • GAB B

    SAÍMOS = I COMO SEGUNDA VOGAL DO HIATO

    PASTÉIS, LENÇÓIS = OXÍTONAS DIT. ABERTO


ID
5338030
Banca
Aeronáutica
Órgão
EEAR
Ano
2021
Provas
Disciplina
Português
Assuntos

Leia:

“Sua alegria povoava o mundo de sorrisos, e esse mundo festivo não só continuava mas também se alargava em seus sonhos e meditações.”

No período composto por coordenação acima, há

Alternativas
Comentários
    • Só temos duas conjunções sindéticas aditivas são elas o "E" e o "Mas também"

    @maurofilho87

  • “Sua alegria povoava o mundo de sorrisos, e esse mundo festivo não só continuava mas também se alargava em seus sonhos e meditações.”

    As palavras destacadas são duas conjunções aditivas, por isso teremos duas orações coordenadas sindéticas aditivas.

    GABARITO: LETRA A

    Me acompanhe lá no YouTube, onde tenho diversas resoluções de questões ↙

    https://www.youtube.com/c/ConcurseirodeElite

  • vírgula antes do e não seria adversativa? não compreendi

  • Columbine, eu também estava com essa dúvida, mas analisando melhor, acho que a vírgula está ali para marcar a mudança do sujeito. Na 1º temos - Sua alegria; Na segunda - mundo festivo

    Obs: posso estar equivocada!

  • Não pode generalizar Columbine, tem exceções infelizmente :)

  • Oliveira vi e uma aula em que o (e) a vírgula pode ser usada ou não.
  • Errei por falta de atenção, fui contando as conjunções e não me atentei de que o último ''e'' não fazia parte de uma oração. Que ódio!


ID
5338033
Banca
Aeronáutica
Órgão
EEAR
Ano
2021
Provas
Disciplina
Português
Assuntos

Assinale a alternativa que apresenta o correto significado da palavra, considerando-se o prefixo destacado.

Alternativas
Comentários
  • Só uma curiosidade todos esses prefixos tirados do livro do Cegalla pág 111 e 112 kkkkkkkkk, vamos para questão

    a) inframencionado: mencionado acima

    Errada, O prefixo Infra de acordo com o Cegalla significa Abaixo, na Parte Superior

    b) anteclássico: contrário ao clássico

    Errada, Ante de acordo com o Cegalla significa Antes, Anterioridade

    c) introspectivo: voltado para fora

    Errada, Intro de acordo com o Cegalla significa Dentro

    d) postergar: deixar para depois

    Correta, de acordo com o Cegalla significa atrás, depois

    @maurofilho87

  • anteclássico = que precede os clássicos ou o classicismo; pré-clássico (ante) DE ANTES

    anticlássico = que se opõe ao classicismo ou ao que é clássico. (anti) PREFIXO DE NEGAÇÃO.


ID
5338036
Banca
Aeronáutica
Órgão
EEAR
Ano
2021
Provas
Disciplina
Português
Assuntos

Coloque (PO) para predicativo do objeto e (PS) para predicativo do sujeito. Em seguida, assinale a alternativa com a sequência correta.

( ) A finalização da pintura resultou magnífica.
( ) A doença o deixou irreconhecível.
( ) As duas mulheres entraram no recinto sérias.
( ) Achavam-no um gênio.

Alternativas
Comentários
  • (PS) A finalização da pintura resultou magnífica

    Predicativo do Suj, pois perceba que magnífica é a característica da finalização da pintura

    (PO) A doença o deixou irreconhecível.

    Predicativo do Obj, irreconhecível não é a característica da doença é característica dele, logo P.O

    (PS) As duas mulheres entraram no recinto sérias.

    Predicativo do Suj,se pergunte quem entraram no recinto Sérias? Automaticamente você irá responder As duas Mulheres , logo se trata de um P.S

    (PO) Achavam-no um gênio.

    Predicativo do Obj, gênio não é a característica de quem achava portanto só pode ser P.O

    @maurofilho87

  • A primeira oração eu achei melhor tentar colocar em ordem direta, eu bati os olhos e percebi que era uma " pegadinha ", pois se você colocar em ordem direta, fica assim: A pintura resultou em uma finalização magnífica. Agora se pergunte qual papel o adjetivo está fazendo... Isso mesmo, predicativo do sujeito.

  • Leandro, mas nesse caso "magnífica não estaria caracterizando a finalização?

    resolvi da seguinte maneira: troquei o "resultou" pelo "ficou. Quem ficou magnífica? A pintura.


ID
5338039
Banca
Aeronáutica
Órgão
EEAR
Ano
2021
Provas
Disciplina
Português
Assuntos

Assinale a alternativa que apresenta, em destaque, adjunto adnominal e adjunto adverbial.

Alternativas
Comentários
  • a) Símbolo da riqueza terrestre, o ouro nasce no espaço.

    Terrestre- Um adjetivo, logo podemos classifica-lo como Adj Adn

    No espaço- Adj Advérbial de lugar

    • As outras assertivas tem adj adverbial porém não tem Adj Adn se trata de Suj

    OBS: Caso tenha erro pode corrigir :)

    @maurofilho87

  • Terrestre = Da terra

    ideia de posse = Adn.

  • Alguém explica o erro da B?

  • Na alternativa B, a palavra legendária é um adjunto adnominal. Todavia, o erro dela está ao afirmar que do Havaí é adjunto adverbial, sendo que este é também adjunto adnominal . Nesse caso, pensem no sentido ativo e passivo

    as ondas são legendárias -> sentido ATIVO ( adjunto adnominal )

    o Havaí tem ondas -> sentido ATIVO ( caso de posse, adjunto adnominal )

  • ondas = substantivo concreto

    do Havaí = ADN


ID
5338042
Banca
Aeronáutica
Órgão
EEAR
Ano
2021
Provas
Disciplina
Português
Assuntos

Assinale a alternativa que contém a pontuação correta do texto.

Alternativas
Comentários
  • a) Os cinco sentidos do homem, ajudam-no a tomar conhecimento de tudo que, acontece à sua volta, seja um ruído, seja uma lâmpada acesa, um odor; qualquer coisa pode atuar sobre ele como um estímulo capaz de provocar uma associação significativa.

    Errada, a primeira vírgula tá errada pois tá separando o Sujeito do Verbo e isso não pode acontecer

    b) Os cinco sentidos do homem ajudam-no a tomar conhecimento de tudo que acontece à sua volta, seja um ruído, seja uma lâmpada acesa, um odor. Qualquer coisa pode atuar sobre ele como um estímulo capaz de provocar uma associação significativa.

    Correta, colocamos vírgula Pois ta listando e em " Um odor" colocamos virgular para indicar uma elipse

    c) Os cinco sentidos do homem ajudam-no a tomar conhecimento, de tudo que acontece à sua volta, seja um ruído, seja uma lâmpada acesa, um odor: qualquer coisa pode atuar sobre ele, como um estímulo capaz de provocar uma associação significativa.

    Errada, não colocamos vírgula entre verbo e complemento

    d) Os cinco sentidos do homem, ajudam-no a tomar conhecimento de tudo que acontece à sua volta seja um ruído seja uma lâmpada acesa, um odor. Qualquer coisa pode atuar sobre ele como um estímulo, capaz de provocar, uma associação significativa.

    Errada, a primeira vírgula tá errada pois tá separando o Sujeito do Verbo e isso não pode acontecer

    @maurofilho87

  • Esta questão avalia o uso da pontuação e, mais especificamente, o uso da vírgula. Antes de procedermos com a análise de cada alternativa, é importante lembrar que os termos essenciais (sujeito e predicado) e integrantes (complemento nominal, objeto direto, objeto indireto e agente da passiva) da oração estão ligados diretamente e, portanto, não apresentam uma pausa. A vírgula não deve ser empregada consequentemente.

    Vejamos qual alternativa apresenta sequência sem nenhum desvio em relação ao uso dos sinais de pontuação.

    A) Os cinco sentidos do homem, ajudam-no a tomar conhecimento de tudo que, acontece à sua volta, seja um ruído, seja uma lâmpada acesa, um odor; qualquer coisa pode atuar sobre ele como um estímulo capaz de provocar uma associação significativa.
    Incorreto. Analisando a alternativa, vemos que “Os cincos sentidos do homem" é o sujeito da oração. Como apontado acima, não se separa o sujeito do predicado por vírgula. Logo, essa alternativa pode ser descartada de antemão.

    B) Os cinco sentidos do homem ajudam-no a tomar conhecimento de tudo que acontece à sua volta, seja um ruído, seja uma lâmpada acesa, um odor. Qualquer coisa pode atuar sobre ele como um estímulo capaz de provocar uma associação significativa.
    Correto. As duas primeiras vírgulas foram corretamente empregadas separando a conjunção alternativa seja / seja. A terceira vírgula indica a supressão da palavra seja. O ponto final empregado após odor indica a finalização do período. 

    C) Os cinco sentidos do homem ajudam-no a tomar conhecimento, de tudo que acontece à sua volta, seja um ruído, seja uma lâmpada acesa, um odor: qualquer coisa pode atuar sobre ele, como um estímulo capaz de provocar uma associação significativa.
    Incorreto. Nessa alternativa, a primeira vírgula está separando o complemento do seu nome. Conhecimento de quê? De tudo o que acontece à sua volta. Como os termos integrantes devem ser ligados diretamente, a vírgula não deve ser empregada separando o complemento nominal.

    D) Os cinco sentidos do homem, ajudam-no a tomar conhecimento de tudo que acontece à sua volta seja um ruído seja uma lâmpada acesa, um odor. Qualquer coisa pode atuar sobre ele como um estímulo, capaz de provocar, uma associação significativa.
    Incorreto. O mesmo desvio da letra A é reproduzido aqui: a separação entre o sujeito e o predicado. Além desse desvio, há separação também entre o verbo e o seu complemento no segundo período - provocar uma associação significativa.


    Gabarito da Professora: Letra B.

ID
5338045
Banca
Aeronáutica
Órgão
EEAR
Ano
2021
Provas
Disciplina
Português
Assuntos

Avalie as afirmações entre parênteses sobre os substantivos em destaque.

I- Das toalhas do enxoval não se via mais sinal do bordado. (Coletivo: conjunto de objetos de noivas, de estudantes, etc.)
II- Ver a pequenina borboleta era símbolo de casamento à vista. (Sobrecomum: usado somente no feminino.)
III- Para o ator, interpretar cada personagem é sempre um desafio. (Classifica-se como masculino e feminino.)
IV- O público tem glamorizado os vilões de novelas. (Outra forma de plural é vilãos.)

   Está correto o que se afirma em 

Alternativas
Comentários
  • A II é epiceno!
  • I- Das toalhas do enxoval não se via mais sinal do bordado. (Coletivo: conjunto de objetos de noivas, de estudantes, etc.)

    Correta, esse é o real significado do coletivo enxoval

    II- Ver a pequenina borboleta era símbolo de casamento à vista. (Sobrecomum: usado somente no feminino.)

    Errada, como Alyce falou Substantivo Epiceno pois pra diferenciar falamos Barboleta Macho ou Barboleta fêmea , com animais os subtantivos geralmente são epicenos

    III- Para o ator, interpretar cada personagem é sempre um desafio. (Classifica-se como masculino e feminino.)

    Correta, o substantivo personagem é uniforme pois eu posso falar O personagem ou A personagem

     IV- O público tem glamorizado os vilões de novelas. (Outra forma de plural é vilãos.)

    Correta, esses são os verdadeiros plurais da palavra vilão

    @maurofilho87


ID
5338048
Banca
Aeronáutica
Órgão
EEAR
Ano
2021
Provas
Disciplina
Português
Assuntos

Quanto à concordância verbal, assinale a alternativa que contém a sequência que completa, correta e respectivamente, as lacunas do texto abaixo.

Caros cidadãos:
Amanhã um grupo __________ (passar) nas ruas deste bairro arrecadando roupas para as vítimas da última tempestade. A maioria das peças __________ (dever) ser para adultos; apenas 10% das roupas __________ (ir) para crianças de 2 a 7 anos. _________ (Haver) muitos pedidos de cobertores, por isso podem doar esse item também.
A Associação de Bairro agradece a todos!

Alternativas
Comentários
  • 1° espaço: passará ---> Concorda com "grupo".

    2° espaço: deverá ou deverão ---> Concordam com "a maioria" e "peças", respectivamente.

    3° espaço: irão ---> Concorda com "10%" ou com "roupas".

    4° espaço: houve ---> Verbo haver no sentido de existir. É impessoal. Deve permanecer no singular.

    GABARITO: LETRA A

    • "a maior parte de”, “parte de”, “a maioria de”, “grande número de”..., seguida de substantivo ou pronome no plural,o verbo, quando posposto ao sujeito, pode ir para o singular ou para o plural.

    Ex: A maior parte dos homens respeita/respeitam as diferenças.

    @maurofilho87


ID
5338051
Banca
Aeronáutica
Órgão
EEAR
Ano
2021
Provas
Disciplina
Português
Assuntos

Assinale a alternativa em que há erro de grafia na palavra em destaque.

Alternativas
Comentários
  • Quando o prefixo termina em vogal e o segundo elemento inicia com R ou S não se usa hífen. Nesse caso, R ou S devem ser duplicados. O certo seria: "SacroSSantos".

    Portanto, gabarito letra C)

  • Sacrossantos seria a ortografia correta.
  • Caso o segundo elemento comece com (S) OU (R) o segredo é juntar e dobrar

    Ex: Antes= mini-Saia

    Agora= miniSSaia

  • GABA-C

    --->Se o prefixo termina com vogal e a segunda palavra começa com (R OU S) duplica

    Minissaia

    Antissocial

  • A questão requer conhecimentos ortográficos e sobre regra do hífen.

    Atentar-se ao comando do enunciado, pois se pede a alternativa em que a palavra destacada está com a grafia errada.

    Alternativa (A) - A palavra “gene" está graficamente correta.

    Alternativa (B) - A palavra composta “semi-integrais" está graficamente correta. Usa-se hífen quando o prefixo termina por vogal e o segundo elemento começa pela mesma vogal.

    Alternativa (C) - A palavra composta “sacro-santos" está graficamente incorreta. O certo seria “sacrossantos". Não se usa hífen quando o prefixo termina em vogal e o segundo elemento começa com “r" ou “s". Nesse caso, dobram-se essas consoantes.

    Alternativa (D) - A palavra “prevenir" está graficamente correta.

    De acordo com o comando do enunciado, a alternativa em que há uma palavra com a grafia errada é a (C).

    Gabarito da Professora: Letra C.

  • OS IGUAIS SE SEPARAM: SEMI-Integrais.

    OS DIFERENTES SE JUNTAM: SACROSSANTO, SEMIDEUS

    OBS: Se o prefixo termina com vogal e a segunda palavra começa com (R OU S) ele é duplicado

  • e outra, não sei como se escreve prevenir. LEMBRE SEMPRE DE PREVENÇÃO.

  • Gab: C

    SACROSSANTOS


ID
5338054
Banca
Aeronáutica
Órgão
EEAR
Ano
2021
Provas
Disciplina
Português
Assuntos

Considere as orações subordinadas adverbiais nos versos seguintes.

“Ainda que eu falasse línguas,
as dos homens e dos anjos
se eu não tivesse amor,
seria como sino ruidoso
ou como címbalo estridente.”

Marque a alternativa que apresenta a classificação ausente no trecho.

Alternativas
Comentários
  • Ainda que= Concessão

    ''Se'' eu não= Condicional

    Seria ''como''= comparação

  • OBS: Ele pede aquela conjunção que não aparece no texto, ou seja , como ele fala ausente

    a) Causal

    Correta, no texto não vemos nenhuma conjunção causal

    b) Concessiva

    Errada, temos sim uma conjunção Concessiva no texto a qual se trata do Ainda que

    c) Condicional

    Errada, temos uma conjunção condicional sim quando ele fala "Se eu não tivesse..."

    d) Comparativa

    Errada, temos sim quando ele fala " Como sino ruidoso..."

    Ainda que eu falasse línguas, as dos homens e dos anjos se eu não tivesse amor, seria como sino ruidoso ou como címbalo estridente.”

    @maurofilho87

  • Acertei, porém não acho fácil lembrar estas classificações.

  • Ainda que (concessiva) eu falasse línguas, as dos homens e dos anjos se (condicional) eu não tivesse amor, seria como (comparativa) sino ruidoso ou como címbalo estridente.”

  • Causal: pois, por que, uma vez, porquanto, desde que, porque, visto, já que, que, dado que, na medida em que.


ID
5338057
Banca
Aeronáutica
Órgão
EEAR
Ano
2021
Provas
Disciplina
Português
Assuntos

Assinale a alternativa incorreta quanto à classificação da voz verbal.

Alternativas
Comentários
  • B- O público presente havia aplaudido friamente o palestrante no início do evento. (voz passiva) - INCORRETA

    Voz passiva só ocorre com VERBO TRANSITIVO DIRETO, fazendo o OBJETO DIRETO virar SUJEITO PACIENTE.

    EX.: O corretor vende casas. (voz ativa-sujeito praticando a ação do verbo)

    Casas são vendidas pelo corretor. (voz passiva analítica- sujeito sofrendo ação do verbo)

    Vendem-se casas pelo corretor. (voz passiva sintética- sujeito sofrendo ação do verbo)

  • a) público e o palestrante saudaram-se friamente no início do evento. (voz reflexiva)

    Correta, voz reflexiva recíproca porque saudaram uns aos outros

    b) O público presente havia aplaudido friamente o palestrante no início do evento. (voz passiva)

    Errada, a estrutura da voz passiva seria essas ai que tá embaixo, como não temos nenhuma logo a alternativa se torna errada

    • Passiva Analítica: Suj + Verbo Auxiliar ( Geralmente Ser, Estar e Ficar) + Verbo no Particípio + Agente da Passiva
    • Passiva Sintética( Pronominal) : V.T.D Conjugado na 3º p do sing ou do plural + Pronome Apassivador (SE) + Suj paciente

    c) A saudação do palestrante foi friamente recebida pelo público presente no início do evento. (voz passiva)

    Correta, Perceba que sim temos uma voz passiva e ela é mais analítica pode observar na estrutura acima

    d) O público presente tinha respondido friamente à saudação do palestrante no início do evento. (voz ativa

    Correta, voz ativa é aquela que o suj pratica a ação perceba que o público tá praticando a ação de responder friamente

    @maurofilho87

    OBS: Respondi toda prova de portg da EEAR dá aquela moral lá no insta KKKKKKKKKK


ID
5338060
Banca
Aeronáutica
Órgão
EEAR
Ano
2021
Provas
Disciplina
Inglês
Assuntos

Read the text and answer question.

Shallow
Lady Gaga

Tell me something, girl
Are you happy in this modern world?
Or do you need more?
Is there something else you’re searching for?
I’m falling
In all the good times
I find myself longing for change
And in the bad times I fear myself
Tell me something, boy
Aren’t you tired trying to fill that void?
Or do you need more?
Ain’t it hard keeping it so hardcore?
I’m falling
In all the good times
I find myself longing for change
And in the bad times I fear myself
I’m off the deep end, watch as I dive in
I’ll never meet the ground
Crash through the surface
Where they can’t hurt us
We’re far from the shallow now
 https://www.letras.mus.br/lady-gaga/shallow-feat-bradley-cooper/

The words “longing”, “change”, “myself” and “hardcore”, underlined in the text, are respectively: 

Alternativas
Comentários
    • Perceba que o primeiro se trata de um Verbo pois perceba no texto que Longing tem uma ideia de desejar, perceba no trecho que ele fala que "I find myself longing for change" ( Encontro-me desejando uma mudança),

    • Change funciona como sendo um noun ( Substantivo) , Change tem o significado de mudança

    • Myself , pronome reflexivo lembre se que os pronomes reflexivos tem o Self em sua terminação EX: HimSelf, MySelf

    • Hardcore, seria um adjetivo qualificando que algo é muito difícil

    OBS: Sendo assim concluímos que o gab é D

    @maurofilho87

  • GAB: B

  • GAB: B

  • GAB: B

  • GAB: B

  • GAB: B

  • GAB: B

  • GAB: B

  • GAB: B

  • matei a questão apenas pela regra do gerúndio "ing" e do pronome reflexivo "myself". mesmo sem saber a tradução.

ID
5338063
Banca
Aeronáutica
Órgão
EEAR
Ano
2021
Provas
Disciplina
Inglês
Assuntos

Read the text and answer question.

Shallow
Lady Gaga

Tell me something, girl
Are you happy in this modern world?
Or do you need more?
Is there something else you’re searching for?
I’m falling
In all the good times
I find myself longing for change
And in the bad times I fear myself
Tell me something, boy
Aren’t you tired trying to fill that void?
Or do you need more?
Ain’t it hard keeping it so hardcore?
I’m falling
In all the good times
I find myself longing for change
And in the bad times I fear myself
I’m off the deep end, watch as I dive in
I’ll never meet the ground
Crash through the surface
Where they can’t hurt us
We’re far from the shallow now
 https://www.letras.mus.br/lady-gaga/shallow-feat-bradley-cooper/

The word “void”, in bold in the text, is closest in meaning to

Alternativas
Comentários
    • Ele quer saber qual assertiva é um significado mais próximo dá palavra Vazio ( Void)

    a) meaningful (significativo)

    Errada, não tem nada a ver com a palavra

    b) emptiness(Vazio)

    Correta, essa é um dos sinônimos para void, vou listar outros para criar vocabulário : Blank,Blankess,Vacuum,Vacuity

    c) fullness(plenitude)

    Errada, não tem nada a ver com a palavra

    d) solid(sólido)

    Errada, não tem nada a ver com a palavra

    OBS: Infelizmente esse tipo de questão é vocabulário :(

    @maurofilho87


ID
5338066
Banca
Aeronáutica
Órgão
EEAR
Ano
2021
Provas
Disciplina
Inglês
Assuntos

Read the text and answer question.

The World’s strangest law

1. You can’t call a pig Napoleon in France.
2. It is illegal to die in the Houses of Parliament.
3. In Miami, Florida, you mustn’t skateboard in a police station.
4. In London, you don’t have to pay to take a flock of sheep across London Bridge.
5. In Florida, unmarried women can’t parachute on Sundays.
6. It’s illegal to play golf on the streets of New York.
7. In Kentucky the law still says that everyone must have a bath at least once a year.
8. In seventeenth-century Russia, you couldn’t grow a beard unless you paid a special tax.
9. In fifteenth-century England, it was illegal for men to wear a moustache.
10. In the USA in the eighteenth century, bars couldn’t sell soda water on Sundays. 

From the book Practical Grammar John Hughes and Ceri Jones.

According to the text, choose the correct alternative:

Alternativas
Comentários
  • a) In Kentucky, the law says that everyone should have a bath at least once a year.( Em Kentucky, a lei diz que todos devem tomar banho pelo menos uma vez por ano.)

    Errada, da alternativa está no Should porque lá no texto tem o Must , lembrando que Must significa( Proibição , Obrigação) e Should significa(Um conselho) é como se na assertiva ele falasse " é você pode tomar banho uma vez no ano mas não é obrigado" e no texto fala que você deve tomar banho pelo menos uma vez no ano

    b) It’s allowed to play soccer on the streets of New York(É permitido jogar futebol nas ruas de Nova York)

    Errada, essa assertiva vai além do texto, pois no texto ele afirma que não pode jogar golfe nas ruas de New York ai na assertiva ele fala que não é permitido jogar futebol nas ruas de New York ; eu não posso afirmar isso pq o texto não fala nada sobre isso o texto só fala que é proibido golfe ele não fala que outros esportes sem ser golfe é permitido

    c) In the USA, bars must sell soda water on Sundays.(Nos EUA, os bares devem vender água com gás aos domingos)

    Errada, o erro também está no Must pq na assertiva ele fala que os bares devem vender, e no texto fala " couldn’t sell" ( Ou seja eles não podem vender ) , a assertiva tá justamente o oposto falando que ele pode vender

    d) In France, it is not allowed to call a pig Napoleon(Na França, não é permitido chamar um porco de Napoleão)

    Correta, é justamente o que o texto fala pode perceber no 1

    @maurofilho87


ID
5338069
Banca
Aeronáutica
Órgão
EEAR
Ano
2021
Provas
Disciplina
Inglês

Read the text and answer question.

The Last Kingdom

   The Last Kingdom is a contemporary story of redemption, vengeance and self-discovery set against the birth of England. The series combines real historical figures and events with fiction, retelling the history of King Alfred the Great and his desire to unite the many separate kingdoms into what would become England.
   Set in the 9th century AD, many of the separate kingdoms of what we now know as England have fallen to the invading Vikings, only the great Kingdom of Wessex stands defiant under its visionary King Alfred the Great. It is the last kingdom. Against this turbulent backdrop lives Uhtred. Born the son of a Saxon nobleman, he is orphaned by the Vikings and then kidnapped and raised as one of their own. Forced to choose between the country of his birth and the people of his upbringing, his loyalties are ever tested. What is he — Saxon or Viking? On a quest to claim his birthright, Uhtred must tread a dangerous path between both sides if he is to play his part in the birth of a new nation and, ultimately, recapture his ancestral lands.
    The Last Kingdom is a show of heroic deeds and epic battles but with a thematic depth that embraces politics, religion, warfare, courage, love, loyalty and our universal search for identity. Combining real historical figures and events with fictional characters, it is the story of how a people combined their strength under one of the most iconic kings of history in order to reclaim their land for themselves and build a place they call home.
Adapted from https://www.bbcamerica.com/shows/the-last-kingdom/about 

In the fragment “Set in the 9th century AD, many of the separate kingdoms of what we now know as England have fallen to the invading Vikings, only the great Kingdom of Wessex stands defiant under its visionary King Alfred the Great”, the adjective pronoun (“its”) refers to:

Alternativas

ID
5338072
Banca
Aeronáutica
Órgão
EEAR
Ano
2021
Provas
Disciplina
Inglês
Assuntos

Read the text and answer question.

The Last Kingdom

   The Last Kingdom is a contemporary story of redemption, vengeance and self-discovery set against the birth of England. The series combines real historical figures and events with fiction, retelling the history of King Alfred the Great and his desire to unite the many separate kingdoms into what would become England.
   Set in the 9th century AD, many of the separate kingdoms of what we now know as England have fallen to the invading Vikings, only the great Kingdom of Wessex stands defiant under its visionary King Alfred the Great. It is the last kingdom. Against this turbulent backdrop lives Uhtred. Born the son of a Saxon nobleman, he is orphaned by the Vikings and then kidnapped and raised as one of their own. Forced to choose between the country of his birth and the people of his upbringing, his loyalties are ever tested. What is he — Saxon or Viking? On a quest to claim his birthright, Uhtred must tread a dangerous path between both sides if he is to play his part in the birth of a new nation and, ultimately, recapture his ancestral lands.
    The Last Kingdom is a show of heroic deeds and epic battles but with a thematic depth that embraces politics, religion, warfare, courage, love, loyalty and our universal search for identity. Combining real historical figures and events with fictional characters, it is the story of how a people combined their strength under one of the most iconic kings of history in order to reclaim their land for themselves and build a place they call home.
Adapted from https://www.bbcamerica.com/shows/the-last-kingdom/about 

According to the text:

Alternativas
Comentários
  • a) Although the series shows many brave actions and battles, it also includes deeper topics(Embora a série mostre muitas ações e batalhas corajosas, também inclui tópicos mais profundos)

    Correta, podemos concluir que inclui alguns tópicos mais profundos quando ele fala " a thematic depth that embraces politics, religion, warfare, courage, love, loyalty and our universal search for identity" ( uma profundidade temática que abrange política, religião, guerra, coragem, amor, lealdade e nossa busca universal pela identidade), perceba que não só se trata de batalhas, ações ; trata-se também de amor; religião( Aprofundando assim o tema)

    b) The main character, Uhtred, is often tested, which makes the series repetitive and in need of identity(O personagem principal, Uhtred, é frequentemente testado, o que torna a série repetitiva e com necessidade de identidade)

    Errada, não se restringe apenas a isso

    c) When combining real events with fiction, the series only real strength lies in the character of King Alfred the Great( Ao combinar eventos reais com ficção, a única força real reside no personagem do Rei Alfred, o Grande)

    Errada, não é a única força

    d) The show focuses on great battles and lacks depth when representing the characters. ( O programa foca em grandes batalhas e carece de profundidade na representação dos personagens)

    Errada, ele descreve os personagens exemplo mesmo é o King Alfred 

    OBS: Questão de Inglês sempre procura a assertiva sempre com mais informações sobre o texto

    @maurofilho87

  • Eu quase acertei essa questão porém minha decisão ficou entre a A e C.


ID
5338075
Banca
Aeronáutica
Órgão
EEAR
Ano
2021
Provas
Disciplina
Inglês
Assuntos

Read the text and answer question.

The Last Kingdom

   The Last Kingdom is a contemporary story of redemption, vengeance and self-discovery set against the birth of England. The series combines real historical figures and events with fiction, retelling the history of King Alfred the Great and his desire to unite the many separate kingdoms into what would become England.
   Set in the 9th century AD, many of the separate kingdoms of what we now know as England have fallen to the invading Vikings, only the great Kingdom of Wessex stands defiant under its visionary King Alfred the Great. It is the last kingdom. Against this turbulent backdrop lives Uhtred. Born the son of a Saxon nobleman, he is orphaned by the Vikings and then kidnapped and raised as one of their own. Forced to choose between the country of his birth and the people of his upbringing, his loyalties are ever tested. What is he — Saxon or Viking? On a quest to claim his birthright, Uhtred must tread a dangerous path between both sides if he is to play his part in the birth of a new nation and, ultimately, recapture his ancestral lands.
    The Last Kingdom is a show of heroic deeds and epic battles but with a thematic depth that embraces politics, religion, warfare, courage, love, loyalty and our universal search for identity. Combining real historical figures and events with fictional characters, it is the story of how a people combined their strength under one of the most iconic kings of history in order to reclaim their land for themselves and build a place they call home.
Adapted from https://www.bbcamerica.com/shows/the-last-kingdom/about 

Mark the alternative that LACKS the correct synonym for the underlined word.

Alternativas
Comentários
    • A questão pede a assertiva que contenha um sinônimo correto

    a) On a quest to claim his birthright, Uhtred must tread a dangerous path (...) – walk

    Errada, tread significa piso e o walk caminhar, logo nada a ver

    b) (...) only the great Kingdom of Wessex stands defiant under its visionary King Alfred the Great – refusing to obey

    Errada, defiant seria desafiador e refusing to obey seria como fosse se recusando a obedecer, também nada a ver

    c) The Last Kingdom is a contemporary story of redemption, vengeance and self-discovery set against the birth of England. – revenge

    Errada, não bate com o sentido da assertiva quando substituo

    d) (...) it is the story of how a people combined their strength under one of the most iconic kings of history in order to reclaim their land for themselves and build a place they call home. – give back

    Correta, Reclaim seria como fosse recuperar e give back é um sinônimo que cabe na assertiva mantém o sentido

    @maurofilho87

  • Essa foi na intuição

  • Mauro, na verdade é ao contrário, LACKS significa falta, ou seja, ele quer a opção que falta o sinônimo correto, a opção incorreta, e é a letra D porque reclaim significa recuperar, pegar de volta e give back é devolver

    eles não querem devolver nada, querem tomar de volta o que é deles, por isso alternativa D está incorreta. E é a nossa opção.

    As outras estão todas corretas.


ID
5338078
Banca
Aeronáutica
Órgão
EEAR
Ano
2021
Provas
Disciplina
Inglês

Choose T for true and F for false. Then, choose the alternative which corresponds to the correct sequence.

( ) We always go home by bus or by car, never by foot.
( ) The tourists will leave in August 23.
( ) He came home on Sunday afternoon.
( ) She saw him on the end of the line.

Alternativas

ID
5338081
Banca
Aeronáutica
Órgão
EEAR
Ano
2021
Provas
Disciplina
Inglês
Assuntos

Read the text and answer question.

Life on a desert island
Alexander,L.G.

   Most of us have formed an unrealistic picture of life on a desert island. We sometimes imagine a desert island to be a sort of paradise where the sun always shines. Life there is simple and good. Ripe fruit falls from the trees and you never have to work. There is also the other side of the picture: Life on a desert island is wretched - you either starve to death or live like Robison Crusoe waiting for a boat which never comes. Perhaps there is an element of truth in both these pictures, but few of us have had the opportunity to find out.
   Two men who recently spent five days on a coral island whished they had stayed there no longer. They were taking a badly damaged boat from the Virgin Islands to Miami to have it repaired. During the journey, their boat began to sink. They quickly loaded a small rubber dinghy with food, matches, and cans of beer and rowed for a few miles across the Caribbean until they arrived at a tiny coral island. There were hardly any trees on the island and there was no water to drink, but this didn’t prove to be a problem since the men collected rain-water in the rubber dinghy. As they had brought a spear gun with them, they had plenty to eat. They caught lobster and fish every day, and, as one of them put it, “ate like kings”. When a passing tanker rescued them five days later, both men were genuinely sorry that they had to leave. 
New concept English. Developing skills: an integrated course for intermediate students

“Life there is simple and good. Ripe fruit falls from the trees and you never have to work.” These sentences could be connected by the word:

Alternativas
Comentários
    • Life there is simple and good. Ripe fruit falls from the trees and you never have to work.( A vida lá é simples e boa. Frutas maduras caem das árvores e você nunca precisa trabalhar.)

    1. A única assertiva que bate é o because, porque perceba quando substituo as outras ficam o pouco sem sentido

    Criar Vocabulário:

    a) however- porém,contudo

    b)because-porque

    c)despite-Apesar de

    d)though- No entanto

    @maurofilho87

  • Mas uma que acertei na cagada.


ID
5338084
Banca
Aeronáutica
Órgão
EEAR
Ano
2021
Provas
Disciplina
Inglês
Assuntos

Read the text and answer question.

Life on a desert island
Alexander,L.G.

   Most of us have formed an unrealistic picture of life on a desert island. We sometimes imagine a desert island to be a sort of paradise where the sun always shines. Life there is simple and good. Ripe fruit falls from the trees and you never have to work. There is also the other side of the picture: Life on a desert island is wretched - you either starve to death or live like Robison Crusoe waiting for a boat which never comes. Perhaps there is an element of truth in both these pictures, but few of us have had the opportunity to find out.
   Two men who recently spent five days on a coral island whished they had stayed there no longer. They were taking a badly damaged boat from the Virgin Islands to Miami to have it repaired. During the journey, their boat began to sink. They quickly loaded a small rubber dinghy with food, matches, and cans of beer and rowed for a few miles across the Caribbean until they arrived at a tiny coral island. There were hardly any trees on the island and there was no water to drink, but this didn’t prove to be a problem since the men collected rain-water in the rubber dinghy. As they had brought a spear gun with them, they had plenty to eat. They caught lobster and fish every day, and, as one of them put it, “ate like kings”. When a passing tanker rescued them five days later, both men were genuinely sorry that they had to leave. 
New concept English. Developing skills: an integrated course for intermediate students

The men on the island didn’t go thirsty because they:

Alternativas
Comentários
    • A pergunta é The men on the island didn’t go thirsty because they( Os homens na ilha não ficaram com sede porque?)

    b) could drink rain-water (poderia beber agua da chuva)

    Correta, só pode ser letra B porque perceba que ele fala no texto " there was no water to drink, but this didn’t prove to be a problem since the men collected rain-water in the rubber dinghy"(não havia água para beber, mas isso não se mostrou um problema, já que os homens coletavam água da chuva no bote de borracha)

    @maurofilho87


ID
5338087
Banca
Aeronáutica
Órgão
EEAR
Ano
2021
Provas
Disciplina
Inglês
Assuntos

In the sentence “You can start a conversation just by saying...”, the opposite of start is:

Alternativas
Comentários
  • OBS: Questão dada é questão matada haha

    • Ele pede a assertiva que é o oposto da palavra Start

    c) Finish

    Correta, só pode ser letra C porque stardt é começar e finish é terminar com isso Gab c

    Pra Criar Vocabulário:

    a) Purpose- Propósito

    b) Initate-Iniciar

    c) Over-sobre

    @maurofilho87


ID
5338090
Banca
Aeronáutica
Órgão
EEAR
Ano
2021
Provas
Disciplina
Inglês
Assuntos

Read the text and answer question.

Memories
Maroon 5
Chorus:
Here’s to the ones that we got
Cheers to the wish you were here, but you’re not
‘Cause the drinks bring back all the memories
Of everything we’ve been through
Toast to the ones here today
Toast to the ones that we lost on the way
‘Cause the drinks bring back all the memories
And the memories bring back, memories bring back you
There’s a time that I remember, when I did not know no pain
When I believed in forever, and everything would stay the same
Now my heart feel like December when somebody say your name
‘Cause I can’t reach out to call you, but I know I will one day, yeah
Everybody hurts sometimes
Everybody hurts someday,
But everything gon’ be alright
Go and raise a glass and say,
Chorus
There’s a time that I remember when I never felt so lost
When I felt all of the hatred was too powerful to stop (ooh, yeah)
Now my heart feel like an ember and it’s lighting up the dark
I’ll carry these torches for ya that you know I’ll never drop, yeah
Everybody hurts sometimes
Everybody hurts someday,
But everything gon’ be alright
Go and raise a glass and say,
Chorus
www.lyricfind.com.

Choose the alternative that contains a word from the song meaning “a piece of wood or coal, etc. that continues to burn after a fire has no more flames”.

Alternativas
Comentários
    • A alternativa pede Choose the alternative that contains a word from the song meaning “a piece of wood or coal, etc. that continues to burn after a fire has no more flames”. ( Escolha a alternativa que contém uma palavra da música que significa “um pedaço de madeira ou carvão, etc. que continua a queimar depois que o fogo não tem mais chamas”.)

    a) Ember (Chamas)

    Correta, essa é a única que contém o que condiz com a alternativa

    OBS: As outras têm nada a ver

    Para Cria vocabulário

    b) hatred-ódio

    c) same-mesmo

    d) rise-subir

    @maurofilho87


ID
5338093
Banca
Aeronáutica
Órgão
EEAR
Ano
2021
Provas
Disciplina
Inglês
Assuntos

Read the text and answer question.

Memories
Maroon 5
Chorus:
Here’s to the ones that we got
Cheers to the wish you were here, but you’re not
‘Cause the drinks bring back all the memories
Of everything we’ve been through
Toast to the ones here today
Toast to the ones that we lost on the way
‘Cause the drinks bring back all the memories
And the memories bring back, memories bring back you
There’s a time that I remember, when I did not know no pain
When I believed in forever, and everything would stay the same
Now my heart feel like December when somebody say your name
‘Cause I can’t reach out to call you, but I know I will one day, yeah
Everybody hurts sometimes
Everybody hurts someday,
But everything gon’ be alright
Go and raise a glass and say,
Chorus
There’s a time that I remember when I never felt so lost
When I felt all of the hatred was too powerful to stop (ooh, yeah)
Now my heart feel like an ember and it’s lighting up the dark
I’ll carry these torches for ya that you know I’ll never drop, yeah
Everybody hurts sometimes
Everybody hurts someday,
But everything gon’ be alright
Go and raise a glass and say,
Chorus
www.lyricfind.com.

Choose the alternative that shows an excerpt from the song that is not grammatically correct:

Alternativas
Comentários
  • Now my heart feel like December when somebody sayS your name

    GAB A


ID
5338096
Banca
Aeronáutica
Órgão
EEAR
Ano
2021
Provas
Disciplina
Inglês
Assuntos

Read the text and answer question.

Ieoh Ming Pei

   Born in 1917, Ieoh Ming Pei grew up in Canton, China. When he was seventeen, he went to the United States to learn about building. As it turned out, Pei became one of the most famous architects of the twentieth century.
  Pei is famous for his strong geometric forms. One of his most controversial projects was his glass pyramid at the Louvre in Paris. The old museum had a lot of problems, but no one wanted to destroy it. Pei had to __________ a solution. Many Parisians were shocked with his proposal for a 71-foot-high glass pyramid. It __________ anyway, blending with the environment. Today many people say that it is a good example of the principles of feng shui.
From the book Grammar Express Marjorie Fuchs and Margaret Bonner

Read this article about the architect Ieoh Ming Pei. Choose the best alternative to complete the text subsequently.

Alternativas
Comentários
  • a) give up / turn out

    Errada, give up não seria algo como desistir e ele não desistiu da solução ele foi atrás dela

    b) went up / pay off

    Errada, went up ai seria como ele tivesse que voltar a uma solução o que não faz muito sentido

    c) Come up/ go back

    Errada, come up tá correta sim na primeira dando a ideia de que ele teve que encontrar uma solução o que tá correta, porém na segunda go back ; back da ideia de voltar algo assim e fala que ele subiu o que não tá correta

    d) come up with / went up

    Correta, como já falei o come up with ele da uma ideia de encontrar uma solução e went up aqui dará ideia de subir , observe: "  It went up anyway, blending with the environment. " ( Subiu de qualquer maneira, misturando-se com o ambiente.)

    @maurofilho87


ID
5338099
Banca
Aeronáutica
Órgão
EEAR
Ano
2021
Provas
Disciplina
Inglês
Assuntos

Read the text and answer question.

The lion and the four bulls
Clarke, M.

  A lion used to walk about a field in which four bulls lived. Many times he tried to attack them, but whenever he came near, they turned their tails toward one another so that whichever way the lion tried to attack, he would have to face the horns of one of them.
   At last, however, the bulls started arguing with each other, and each went off to a different part of the field by himself. Then the lion attacked them one by one and soon had killed all four.
Ann Arbor: The University of Michigan Press, 1996.p.138

In the fragment “At last, however, the bulls...”, the word however means the same as:

Alternativas
Comentários
    • Ele quer a assertiva que possa substituir a conjunção do however

    a) Nevertheless

    Correta, significando No Entanto ; é coerente para substituir However

    b) Furthermore

    Errada, dar ideia de adição

    c) In addtion

    Errada, aqui é adição pira

    d) Therefore

    Errada, aqui se trata de uma conjunção conclusiva ; significando portanto

    @maurofilho87


ID
5338102
Banca
Aeronáutica
Órgão
EEAR
Ano
2021
Provas
Disciplina
Inglês
Assuntos

Read the text and answer question.

The lion and the four bulls
Clarke, M.

  A lion used to walk about a field in which four bulls lived. Many times he tried to attack them, but whenever he came near, they turned their tails toward one another so that whichever way the lion tried to attack, he would have to face the horns of one of them.
   At last, however, the bulls started arguing with each other, and each went off to a different part of the field by himself. Then the lion attacked them one by one and soon had killed all four.
Ann Arbor: The University of Michigan Press, 1996.p.138

The messsage of the text can be summarized as:

Alternativas
Comentários
  • a) Necessity is the mother of invention(Necessidade é a mãe da invenção)

    Errada, ele não fala em nenhum momento do texto que precisa de intervenção da mãe

    b) You must take the bull by the horns.(Você deve pegar o touro pelos chifres.)

    Errada, ele em nenhum momento fala que o ataque é assim

    c) United we stand, divided we fall (Unidos nós resistimos, divididos nós caímos)

    Correta, é justamente essa a mensagem que o texto que passar que quando os touros estavam juntos o leão atacaria eles porém como eles estavam juntos a união deles consegueria resistir ao leão , porém quando eles brigam e se separam o leão ataca facilmente um por um matando-o todos , ou seja, é aquela velha frase " A união faz a força"

    d) Only the strong survive(Só os fortes sobrevivem)

    Errada, a mensagem que o texto passa não é essa e sim que juntos somos mais fortes

  • O texto está falando sobre um leão que queria atacar 4 touros, porém eles se juntavam de forma com que o leão teria que enfrentar os chifres de cada um deles. Mas teve um dia que os touros começaram discutir entre eles e acabaram se separando. Então, o leão conseguiu atacar cada um deles.

    GAB LETRA C

    United we stand, divided we fall.

    "Unidos resistimos, divididos caímos"


ID
5338105
Banca
Aeronáutica
Órgão
EEAR
Ano
2021
Provas
Disciplina
Inglês
Assuntos

The sentence that is not correct is:

Alternativas
Comentários
  • Resposta certa letra A, uma vez que não existe more quiet e sim quieter


ID
5338108
Banca
Aeronáutica
Órgão
EEAR
Ano
2021
Provas
Disciplina
Inglês
Assuntos

Read the text and answer question.

Metal airplane part seems to fall from plane into Arizona
family’s backyard

   An Arizona couple discovered what appeared to be a metal plate from an airplane in their backyard last week. Charlie and Jaclyn High of Phoenix found the white metal piece, which had fallen in their backyard, on Friday, CBS5 reported.
   “I kind of looked around to see if there was anything else, like another piece, or something else other than that, with writing on it. It looks like it’s from an airplane, and you think, oh man, that’s crazy,” Jaclyn told the outlet. According to images shared with CBS5, the metal piece seems to be part of the airplane lavatory.
Adapted from https://www.foxnews.com/travel/arizona-metal-piece-airplanebackyard. 

In the sentence “Charlie and Jaclyn High of Phoenix found the white metal piece, which had fallen in their backyard, on Friday, CBS5 reported.”, in bold in the text, choose the alternative that contains a verb connected to the action that happened first.

Alternativas
Comentários
  • Charlie and Jaclyn High of Phoenix found the white metal piece, which had fallen in their backyard, on Friday, CBS5 reported. (Charlie e Jaclyn High of Phoenix encontraram o pedaço de metal branco, que havia caído em seu quintal, na sexta-feira, informou a CBS5.)

    c) Fallen(Caído)

    É a única assertiva que se mantém com o contexto da frase acima

  • fallen = past perfect = pretérito + que perfeito no português


ID
5338111
Banca
Aeronáutica
Órgão
EEAR
Ano
2021
Provas
Disciplina
Inglês
Assuntos

Read the text and answer question.

Metal airplane part seems to fall from plane into Arizona
family’s backyard

   An Arizona couple discovered what appeared to be a metal plate from an airplane in their backyard last week. Charlie and Jaclyn High of Phoenix found the white metal piece, which had fallen in their backyard, on Friday, CBS5 reported.
   “I kind of looked around to see if there was anything else, like another piece, or something else other than that, with writing on it. It looks like it’s from an airplane, and you think, oh man, that’s crazy,” Jaclyn told the outlet. According to images shared with CBS5, the metal piece seems to be part of the airplane lavatory.
Adapted from https://www.foxnews.com/travel/arizona-metal-piece-airplanebackyard. 

The sentence “Charlie and Jaclyn High of Phoenix found the white metal piece, which had fallen in their backyard, on Friday, CBS5 reported.”, contains verbs in the following tenses, in this order:

Alternativas
Comentários
  • a) Simple Past, Past Perfect, Simple Past

    • Found está no simple past é um verbo irregular, sua forma original era find passando para o passado fica found

    • Had Fallen- Tá no Past Perfect, lembre se da estrutura do Past Perfect: Had+ Past Participle

    • Reported - Simple Past , lembrando que verbos regulares no Simple Past só acrescentar o ED no final e verbos irregulares você infelizmente precisa decorar haha

    @maurofilho87


ID
5338114
Banca
Aeronáutica
Órgão
EEAR
Ano
2021
Provas
Disciplina
Inglês
Assuntos

Read the text and answer question.

Metal airplane part seems to fall from plane into Arizona
family’s backyard

   An Arizona couple discovered what appeared to be a metal plate from an airplane in their backyard last week. Charlie and Jaclyn High of Phoenix found the white metal piece, which had fallen in their backyard, on Friday, CBS5 reported.
   “I kind of looked around to see if there was anything else, like another piece, or something else other than that, with writing on it. It looks like it’s from an airplane, and you think, oh man, that’s crazy,” Jaclyn told the outlet. According to images shared with CBS5, the metal piece seems to be part of the airplane lavatory.
Adapted from https://www.foxnews.com/travel/arizona-metal-piece-airplanebackyard. 

In the sentence “Either mom’s cooking dinner or somebody got sick at home.”, the expression either...or gives an idea of:

Alternativas
Comentários
    • Na assertiva o Either or ele funciona como o Ou ,Either mom’s cooking dinner or somebody got sick at home. ( Ou a mamãe está preparando o jantar ou alguém ficou doente em casa. ")

    a) Comparison

    Errada, não tem nada a ver com a frase; tá comparando com nada

    b) Opposition

    Errada, não tem nada a ver com a frase; tá se opondo a nada

    c) Exclusion

    Correta, ocorre sim a exclusão porque Ou ela tá preparando o Jantar Ou alguém tá doente em casa ; é uma coisa ou outra

    d) Additiom

    Errada, tem nenhuma ideia de adição aqui nada a ver

    @maurofilho87

  • Fiz essa questão em sala com um professor ,mas chegamos a conclusão que teria ideia de alternância então a cosideramos anulada

  • todas as correções que vocês fizerem tem que ser com base no livro do edital por incrivel que parivel os livros a maioria deles estão corretos quando pensamos que não ai é barro kkkk mas matêm.

  • A frase deve ser clara e direta, mas não vejo nenhum tipo de exclusão


ID
5338117
Banca
Aeronáutica
Órgão
EEAR
Ano
2021
Provas
Disciplina
Inglês
Assuntos

Read the text and answer question.

Insomnia

   Insomnia is the most common of all sleep complaints. Almost everyone has occasional sleepless nights, perhaps due to stress, heartburn or drinking too much caffeine or alcohol. Insomnia is a lack of sleep that occurs on a regular or frequent basis, often for no apparent reason.
   How much sleep is enough varies. Although 7 1/2 hours of sleep is about average, some people do fine on 4 or 5 hours of sleep. Other people need 9 or 10 hours a night.
  Inability to get a good night’s sleep can affect not only your energy level and mood but your health as well because sleep helps bolster your immune system. Fatigue, at any age, leads to diminished mental alertness and concentration. Lack of sleep is linked to accidents both on the road and on the job.
  About one out of three people have insomnia sometime in their life. Sleeplessness may be temporary or chronic. You don’t necessarily have to live with sleepless nights. Some simple changes in your daily routine and habits may result in better sleep.
htttp://www.mayoclinic.com. 

The text contains information on:

Alternativas
Comentários
  • The text contains information on ( O texto contém informações sobre:)

    A) The different intellectual activities which may keep a person’s mind too busy to sleep. (As diferentes atividades intelectuais que podem manter a mente de uma pessoa ocupada demais para dormir.)

    B) The effects of sleep problems on people’s body and mind.(Os efeitos dos problemas de sono no corpo e na mente das pessoas) - ALTERNATIVA CORRETA

    C) The activities that help promote a good night’s sleep. (As atividades que ajudam a promover uma boa noite de sono.)

    D) The food that is necessary to eat to sleep better. (O alimento que é necessário comer para dormir melhor.)


ID
5338120
Banca
Aeronáutica
Órgão
EEAR
Ano
2021
Provas
Disciplina
Inglês
Assuntos

Read the text and answer question.

Insomnia

   Insomnia is the most common of all sleep complaints. Almost everyone has occasional sleepless nights, perhaps due to stress, heartburn or drinking too much caffeine or alcohol. Insomnia is a lack of sleep that occurs on a regular or frequent basis, often for no apparent reason.
   How much sleep is enough varies. Although 7 1/2 hours of sleep is about average, some people do fine on 4 or 5 hours of sleep. Other people need 9 or 10 hours a night.
  Inability to get a good night’s sleep can affect not only your energy level and mood but your health as well because sleep helps bolster your immune system. Fatigue, at any age, leads to diminished mental alertness and concentration. Lack of sleep is linked to accidents both on the road and on the job.
  About one out of three people have insomnia sometime in their life. Sleeplessness may be temporary or chronic. You don’t necessarily have to live with sleepless nights. Some simple changes in your daily routine and habits may result in better sleep.
htttp://www.mayoclinic.com. 

According to the text, we can NOT infer that:

Alternativas
Comentários
  • OBS: Precisava nem ler o texto creio eu pq todo mundo sabe ou deveria saber o que é insônia a partir dai só eliminar as assertivas e matar a questão :) e lembre se ele quer a errada

    a) Changing some everyday habits can be a step to improve sleep quality(Mudar alguns hábitos diários pode ser um passo para melhorar a qualidade do sono)

    b) A large number of people suffer from occasional sleepless nights.(Um grande números de pessoas sofre de noites sem dormir ocasionais)

    C) Difficulty in concentration can be a consequence of lack of sleep.(A dificuldade de Concentração pode ser consequência da falta de sono)

    D) Insomnia will necessarily lead to chronic sleepless nights. (A insônia necessariamente levará noites crônica sem dormir)

    Errada, ele fala no texto " You don’t necessarily have to live with sleepless nights."(Você não precisa necessariamente viver com noites sem dormir.)

    @MAUROFILHO87

  • Ele quer a alternativa incorreta, logo:

    GAB LETRA D

    De acordo com o texto: "Sleeplessness may be temporary or chronic. You don’t necessarily have to live with sleepless nights." Traduzindo: Insônia PODE ser temporária ou crônica. Você não necessariamente precisa viver com noites de insônia.

    A alternativa D diz o contrário disso, "Insomnia will necessarily lead to chronic sleepless nights." Traduzindo:

    Insonia necessariamente leva a noites cronicas de insonia.


ID
5338123
Banca
Aeronáutica
Órgão
EEAR
Ano
2021
Provas
Disciplina
Inglês
Assuntos

“Why do bees fuss about so much when they fly?”. The singular of this sentence is:

Alternativas
Comentários
  • Why do bees fuss about so much when they fly?”. ( Por que as Abelhas se preocupam tanto quando voam?)

    a) Why do an bee fusses about so much when it flies?

    Errada, esse an não pode tá ai An usmamos com vogal ou som de vogal EX: An Apple

    b) Why does a bee fusses about so much when it flie?

    Errada, o verbo fica Filies porque corta o Y e acrescenta o ies

    c) Why does a bee fuss about so much when it flies?

    Correta, esse a ainda vai ser o meu uma no portg , ou seja vai ficar (Por que uma abelha se preocupa tanto quando voa?)

    d) Why does bee fuss about so much when it flies?

    Errada , para ir para o singular preciso da presença do artigo ; como não tem tá fora

    @maurofilho87

  • GAB LETRA C

    Why does a bee fuss about so much when it flies?

    Como ele quer no singular o does precia vir antes e o verbo não pode ter o "es" no final já que o does já está na frase


ID
5338126
Banca
Aeronáutica
Órgão
EEAR
Ano
2021
Provas
Disciplina
Inglês
Assuntos

Read the text and answer question.

‘Emily in Paris’ star says he partly understands why critics
panned the ‘cliché’ Netflix show

   Despite being a huge hit for Netflix, critics across the board (particularly French critics) have slammed the show for indulging in outdated and offensive stereotypes that present Parisians as rude, sexist, and elitist.
  The main love interest in Netflix’s controversial comedy “Emily in Paris” said he partly understands why critics have panned the show. “I think they’re right in a way,” Lucas Bravo, who plays chef Gabriel in the show, said during an interview with Cosmopolitan.
   The 32-year-old French actor continued: “At some point, if you want to tell a story about Paris, you have to choose an angle. You have to choose a vision. French critics, they didn’t understand the fact that it’s just one vision. They’re like, ‘Oh, this is not what Paris is.’ Of course. Paris is many things.”
Adapted from https://www.insider.com/emily-in-paris-star-lucas-bravounderstands-netflix-show-criticism-2020-10. 

In the sentence “At some point, if you want to tell a story about Paris, you have to choose an angle”, the words “have to” could be substituted by:

Alternativas
Comentários
  • Must = have to, need to >> expressa obrigação ou dever (você deve.., você tem que..,você precisa fazer.. )

    gabarito B

    1. In the sentence “At some point, if you want to tell a story about Paris, you have to choose an angle”, the words “have to” could be substituted by:

    (Na frase “Em algum momento, se você quiser contar uma história sobre Paris, você tem que escolher um ângulo”, as palavras “tenho que” poderiam ser substituídas por:)

    • A) Can (Posso)

    • B) Must (Deve) - Alternativa correta

    • C) Could (Poderia)

    • D) Should (Deve)

  • Have to tem o sentido de obrigação, então:

    • Can tem o sentido de possibilidade x
    • Must tem tem sentido de obrigação
    • Could pode ser o Can no passado ou então, ter o sentido de possibilidade porém mais formal x
    • Should tem o sentido de dever, não de obrigação x

    GAB LETRA B

  • Have to pode substituir o modal must na idéia de obrigação


ID
5338129
Banca
Aeronáutica
Órgão
EEAR
Ano
2021
Provas
Disciplina
Inglês
Assuntos

Read the text and answer question.

‘Emily in Paris’ star says he partly understands why critics
panned the ‘cliché’ Netflix show

   Despite being a huge hit for Netflix, critics across the board (particularly French critics) have slammed the show for indulging in outdated and offensive stereotypes that present Parisians as rude, sexist, and elitist.
  The main love interest in Netflix’s controversial comedy “Emily in Paris” said he partly understands why critics have panned the show. “I think they’re right in a way,” Lucas Bravo, who plays chef Gabriel in the show, said during an interview with Cosmopolitan.
   The 32-year-old French actor continued: “At some point, if you want to tell a story about Paris, you have to choose an angle. You have to choose a vision. French critics, they didn’t understand the fact that it’s just one vision. They’re like, ‘Oh, this is not what Paris is.’ Of course. Paris is many things.”
Adapted from https://www.insider.com/emily-in-paris-star-lucas-bravounderstands-netflix-show-criticism-2020-10. 

The word “panned”, in bold in the text, could NOT be replaced by:

Alternativas
Comentários
  • "The main love interest in Netflix’s controversial comedy “Emily in Paris” said he partly understands why critics have panned the show. “I think they’re right in a way,” Lucas Bravo, who plays chef Gabriel in the show, said during an interview with Cosmopolitan."

    TRADUZINDO:

    "O principal interesse amoroso na polêmica comédia da Netflix, "Emily in Paris", disse que em parte entende por que os críticos criticaram o show. “Acho que eles estão certos de certa forma”, disse Lucas Bravo, que interpreta o chef Gabriel no programa, durante uma entrevista para a Cosmopolitan."

    Questão:

    The word “panned”, in bold in the text, could NOT be replaced by:(A palavra “criticaram”, em negrito no texto, NÃO poderia ser substituída por:)

    A) condemned (condenado)

    B) criticized (criticado)

    C) attacked (atacado)

    D) praised (elogiado) - Resposta certa


ID
5338132
Banca
Aeronáutica
Órgão
EEAR
Ano
2021
Provas
Disciplina
Matemática
Assuntos

Uma caixa cúbica, de aresta 10 cm, está totalmente cheia de água. Ao despejar toda a água num tubo cilíndrico de 5 cm de raio, essa água atingirá a altura de ____/π cm no tubo. (Considere as dimensões como sendo internas aos recipientes e que o tubo tem a altura necessária para o evento.)

Alternativas
Comentários
  • Gabarito (B)

    • Dados

    aresta do cubo = 10

    raio do cilindro = 5

    • Cálculos

    1°) Encontrar o volume do cubo.

    v = a³

    v = 10³

    v = 1000

    2°) Encontrar o volume do cilindro.

    v = π r² h

    v = π 5² h

    v = 25π h

    3°) Igualar e correr para o abraço!

    25π h = 1000

    h = 1000 / 25π

    h = 40 / π

    Bons estudos!


ID
5338138
Banca
Aeronáutica
Órgão
EEAR
Ano
2021
Provas
Disciplina
Matemática
Assuntos

A revolução de um triângulo equilátero, de 6 cm de lado, em torno de um de seus lados, gera um sólido de volume igual a ____________ π cm3 .

Alternativas
Comentários
  • Resolução:

    https://www.youtube.com/watch?v=Zzk2_GHZxU4

  • Teorema de pappus gudim, deem uma estudada sobre.... bem legal, adianta o lado bem


ID
5338141
Banca
Aeronáutica
Órgão
EEAR
Ano
2021
Provas
Disciplina
Matemática
Assuntos

O ponto P(1, 4) é _______________ à circunferência de equação (x + 1)2 + (y − 5)2 = 9 e é _______________ à circunferência de equação (x − 3)2 + (y − 5)2 = 16.

Alternativas
Comentários
  • Apenas substitua cada ponto na equação e veja se esse ponto é interno ou externo através do raio.

    se d>raio, então é externo

    se d<raio, então é interno

    se d= raio, o ponto pertence á circunferência

    P1-> (1+1)² + (4-5)² => 4 + 1 = 5, 5 <9 se d<r = INTERNO

    P2-> ( 1-3)² + (4-5)² => 4 + 1 = 5 , 5 < 16 se d<r = INTERNO

    GABARITO D

  • Questão para ngm zerar.


ID
5338144
Banca
Aeronáutica
Órgão
EEAR
Ano
2021
Provas
Disciplina
Matemática
Assuntos

Em uma classe da 1ª série do Curso de Formação de Sargentos - EEAR, as idades dos alunos se distribuíam conforme a tabela. Desta forma, a idade média ponderada desses alunos era de _______ anos. 

Idade (anos)    18    19    20    21    22  
   fr (%)        40    30    17    10    3

Alternativas
Comentários
  •  18.0,4 + 19.0,3 + 20.0,17 + 21.0,1 + 22.0,03

    7,2 + 5,7 + 3,4 + 2,1 + 0,66 = 19,06

  • (18.40+19.30+20.17+21.10+22.3)/100 = 1906/100 = 19,06 - alternativa C


ID
5338147
Banca
Aeronáutica
Órgão
EEAR
Ano
2021
Provas
Disciplina
Matemática
Assuntos

Sejam A e B os restos das divisões de P(x) = x3 − 3x2 − 4x + 6 por, respectivamente, x + 2 e x − 3. Desta forma, pode-se afirmar que

Alternativas
Comentários
  • como o exercício fala que (x+2) e (x-3) são os divisores, nós podemos achar suas raízes e jogar no polinômio e, por fim, igualar aos respectivos restos. Teorema de Briot-Ruffini

    x+2=0 => x= -2 x-3=0 => x=3

    P(-2)= (-2)^3 - 3( -2)^2 - 4( -2) +6 = A

    P(-2)= 8 - 12 + 8 +6 = A => A= -6

    P(3)= 3^3 - 3(3)^2 -4.3 + 6= B

    P(3)= 27-27-12 +6= B => B= -6

    Portanto A=B

    Gabarito A

  • https://www.youtube.com/watch?v=nhVk2CHsfsw

    esse vídeo tá bem explicado

  • Teorema do resto:

    x+2=0 => x= -2 => P(-2)= (-2)^3 - 3.(-2)^2 - 4.(-2) + 6 = => P(-2)= -6 => A= -6

    x-3=0 => x= 3 => P(3)= (3)^3 - 3.(3)^2 - 4.3 + 6 => P(3) = -6 => B= -6

    A=B= -6


ID
5338150
Banca
Aeronáutica
Órgão
EEAR
Ano
2021
Provas
Disciplina
Matemática
Assuntos

Um número complexo z tem argumento θ = 5π/6 e módulo igual a 6. A forma algébrica de z é

Alternativas
Comentários
  • Alfinha

  • Argumento θ = 5π/6 => θ = 5.180º/6 => θ =150º

    Sabendo que a forma algébrica de um complexo pode ser escrito por z= a + bi

    temos que seus argumentos são

    cosθ=a/|z| e senθ=b/|z|

    temos: cos150º=a/6

    -√3/2=a/6 => a= -3√3

    sen150º=b/6

    1/2= b/6 => b=3

    Portanto, z= -3√3 + 3i

    ALTERNATIVA - A

  • Fórmulas:

    a=cosθ.módulo

    b=senθ.módulo

    Sabendo disso,vamos para as informações:

    módulo=6

    θ=5π/6

    π=180°

    Então, θ=5x180/6=>900/6=>150°

    Após isso,aplicamos as fórmulas.

    a=cos150°x 6

    b=sen150° x 6

    cos 150°=- √3/2

    sen 150°= 1/2

    a= -√3/2 x 6= -3√3

    b= 1/2 x 6 = 3

    Temos então:

    Z=-3√3+3i


ID
5338156
Banca
Aeronáutica
Órgão
EEAR
Ano
2021
Provas
Disciplina
Matemática
Assuntos

Sejam os arcos de 480° e −4π/3 rad. No ciclo trigonométrico, esses arcos são tais que ambos estão no

Alternativas
Comentários
  • 480° = 1 volta + 120° ---> 2° quadrante

    −4π/3 rad = -4.180/3 = -240° = 240° no sentido contrário = 120° ---> 2° quadrante

    GABARITO: LETRA B

  • angulos congruentes possuem mesmo valor, nesse caso -240º = 480º, pois estão no mesmo lugar do ciclo trigonométrico.


ID
5338159
Banca
Aeronáutica
Órgão
EEAR
Ano
2021
Provas
Disciplina
Matemática
Assuntos

Seja a P.G. (24, 36, 54, ...). Ao somar o 5º e o 6º termos dessa P.G. tem-se

Alternativas
Comentários
  • a5 + a6 = a1.q^4 + a1.q^5 colocando o a1.q^4 em evidencia

    a5+a6= a1.q^4( 1+q)

    a1= 24

    q= 36/24 = 3/2

    Logo,

    soma= 24.(3/2)^4 ( 1+3/2 )

    soma=24.(81/16).(5/2)

    soma= 3.(81/4).5 = 1215/4

    LETRA B

  • Gabarito (C)

    • Dados

    a1 = 24

    q = 3/2

    Obs.: Para encontrar a razão (q) na P.G basta dividir 36 por 24, ou 54 por 36.

    • O que a questão pede?

    a5 + a6

    • Cálculos

    1°) O valor de a5

    a5 = 24 . (3/2)^4

    a5 = 24 . 81/16

    Simplifica 24 e 16 por 8

    a5 = 3 . 81/2

    a5 = 243/2

    2°) Para encontrar o a6 basta fazer a5 . q ou joga na fórmula do termo geral novamente. Fica a critério do concurseiro.

    a6 = 243 / 2 . 3/2

    a6 = 729/4

    3°) Agora é só somar e correr para o abraço!

    243/2 + 729/4 = 1215/4

    Bons estudos!

  • https://www.youtube.com/watch?v=Be5_TZg0uww

    Vídeo bem explicado

  • Primeiro precisamos encontrar a Razão (q):

    q= a2/a1

    q= 36/24= 3/2

    Agora iremos encontrar o termo pela fórmula do termo geral:

    an= a1 . q ^n-1

    a5= 24. (3/2)⁵-¹

    a5= 24. (3⁴/2⁴)

    a5= 24. 81/16

    Simplificando 24 e 16 por 8:

    a5= 3. 81/2

    a5= 243/2

    Agora iremos encontrar o termo, multiplicando seu termo anterior com a razão:

    a6= a5.q

    a6= 243/2 . 3/2

    a6= 729/4

    Somando o e o termo:

    a5+a6= 243/2 + 729/4

    O mmc entre 2 e 4 é igual a 4

    Multiplicando 4 pelos demoninadores e depois multiplicando os valores pelos numeradores temos:

    a5+a6= 486+729/4

    a5+a6= 1215/4

    Gabarito: C


ID
5338162
Banca
Aeronáutica
Órgão
EEAR
Ano
2021
Provas
Disciplina
Matemática
Assuntos

Seja uma função f: A → B tal que A = {0, 1, 2, 3, 4} e B = ℝ. A alternativa que apresenta todos os pontos de um possível gráfico de f é

Alternativas
Comentários
  • Só a letra B é possível, pois os valores do Domínio(x) estão todos representados e todos podem ter o mesmo valor da Imagem (y), que nesse caso é 0.

  • definição de função

  • É uma questão que precisa dos conhecimentos de definição de uma função afim.

    Somente a letra B é uma alternativa onde seria possível formar um gráfico, pois os valores do Dm(X) e da Im(Y) estão representados na alternativa.

    Nota-se que os valores de Im de todos os pares é igual a zero, assim teremos elementos diferentes do domínio que geram uma única imagem, o que é plenamente possível em uma função "sobrejetora".

  • questão de relações. Está no livro do Gelson Iezzi


ID
5338165
Banca
Aeronáutica
Órgão
EEAR
Ano
2021
Provas
Disciplina
Matemática
Assuntos

A razão entre o perímetro do quadrado circunscrito a uma circunferência de raio 2 cm e o perímetro do quadrado inscrito a essa mesma circunferência é

Alternativas
Comentários
  • Perímetro do quadrado circunscrito

    Se você desenhar uma circunferência de raio 2 dentro de um quadrado, você vai perceber que o lado do quadrado é igual ao diâmetro, ou seja, o lado do quadrado vale L=2R => L=2.2 = 4

    Perímetro do quadrado circunscrito: P= 4.lado => P=4.4= 16

    Perímetro do quadrado inscrito

    Se você desenhar um quadrado dentro de uma circunferência, você vai perceber que a diagonal do quadrado é igual ao diâmetro, ou seja, Diagonal= 2R

    obs: lembre que a diagonal do quadrado é igual a D=L√2

    Assim, L√2=2.2 => L'=2√2

    Perímetro do quadrado inscrito: P'=4.L' => P'=8√2

    RAZÃO DE P/P' = 16/8√2 => √2

    GABARITO D

  • Esclarecendo uma conta do colega acima...

    • Perímetro do quadrado inscrito

    D = L√2

    A diagonal é duas vezes o raio. Já que o raio é 2, a diagonal será 4. Substituindo o valor na fórmula:

    4 = L√2

    L = 4 / √2

    RACIONALIZANDO

    L = 4 / √2 . √2 / √2

    L = 4 √2 / √4

    L = 4 √2 / 2

    Simplifica 4 e 2

    L = 2√2

    Bons estudos!


ID
5338168
Banca
Aeronáutica
Órgão
EEAR
Ano
2021
Provas
Disciplina
Matemática
Assuntos

Se 8 alunos do CFS da EEAR “entrarão em forma” em uma única fila, de maneira que a única restrição seja a de que o aluno mais alto fique no início da fila, então o número de formas diferentes de se fazer essa formação é

Alternativas
Comentários
  • Gabarito: A

    Bom galera, fiz essa questão passo a passo no link abaixo:

    http://sketchtoy.com/70071582

    #ForçaeHonra

  • Gabarito (A)

    Para resolver essa questão basta permutar (análise combinatória) os 7 lugares que restarão, pois a questão nos diz que o aluno mais alto terá que ficar no início da fila. Ou seja, não importa quantas vezes mudem a ordem dos alunos, o mais alto sempre permanecerá fixo sendo o primeiro da fila. Portanto:

    7! = 7.6.5.4.3.2.1 = 5040

    Bons estudos!

  • Tem 8 pessoas que deverão formar uma fila, mas 1 dessas 8 sempre vai ocupar um lugar fixo que não vai ser alterado com a permutação dos demais.

    Assim :

    1/_/_/_/_/_/_/_/

    O restante pode ser permutado pelos restantes dos alunos, que são 7:

    S = 1x7x6x5x4x3x2x1.

    S = 5040

    Letra A)


ID
5338177
Banca
Aeronáutica
Órgão
EEAR
Ano
2021
Provas
Disciplina
Matemática
Assuntos

Se sen 2x = 1/3 então (sec x) : (sen x) é igual a

Alternativas
Comentários
  • Sen (2x) = 2.senx.cosx

    2senx.cosx = 1/3 ---> senx.cosx = 1/6

    Secx/ senx ---> 1/cosx/senx ---> 1/cosx.senx ---> 1/1/6 ---> 6

  • Sen2x = 2SenxCosx = 1/3

    SenxCosx = 1/6

    (SexCosx = 1/6)^(-1) <=> CossecXSecx = 6

    Cossecx = 1/Senx

    Substituindo, temos:

    1/SenxSecx = 6

    Que é o mesmo que: Secx/Senx = 6

  • Fórmula necessária para a resolução da questão:

    Sen (a + b) = sen a * cos b + sen b * cos a

    Sen 2x = 1/3 ; 2 * sen x * cos x = 1/3; sen x * cos x = 1/6

    Sabendo que a secante é o inverso do cosseno, teremos:

    sec x /sen x = 1/cosx * 1/ senx; 1/ (senx * cos x) = 1/ 1/6 = 6

    GABARITO: B

    @simplificandoquestoescombizus (Jefferson Lima)


ID
5338180
Banca
Aeronáutica
Órgão
EEAR
Ano
2021
Provas
Disciplina
Matemática
Assuntos

Pedro é um tenista profissional que vem treinando 120 saques por dia. Porém, a partir de amanhã, a cada dia de treino ele fará 5 saques a mais que no treino anterior. Se o objetivo de Pedro é alcançar o dia em que treinará 180 saques, ele conseguirá isso no ____ dia de treino, considerando hoje o primeiro dia.

Alternativas
Comentários
  • Gabarito (C)

    Por progressão aritmética resolve.

    a1 = 120

    r = 5

    an = 180

    Fórmula do termo geral da P.A

    an = a1 + (n-1) . r

    • Cálculo:

    180 = 120 + (n-1) . 5

    180 - 120 = 5n - 5

    60 = 5n - 5

    60 + 5 = 5n

    65 = 5n

    n = 65/5

    n = 13

    Bons estudos!

  • Eu fiz da seguinte forma:

    120 + 5x = 180

    5x = 180 - 120

    5x = 60

    x = 60/5

    x = 12

    (como na questão diz 'considerando hoje o primeiro dia' somamos +1)

    x = 12 + 1

    x = 13


ID
5338183
Banca
Aeronáutica
Órgão
EEAR
Ano
2021
Provas
Disciplina
Matemática
Assuntos

Simplificando a expressão y = Cn,4/Cn-1,3, encontra-se y igual a

Alternativas
Comentários
  • letra D

  • alguem faz a resolução desta

  • Seria a fórmula de combinação simples dividida por ela própria Cnp= n!/p!(n-p)! / n!/p!(n-p)! substituindo ficará: n!/4!(n-4)! / (n-1)!/ 3!(n-4)! basta resolver a expressão.
  • Vou LOGO AVISANDO NAO É PROPAGANDA eu poderia colocar a resposta mas acredito que ficaria muito confuso então prefiro compartilhar o vídeo que também me ajudou a encontrar uma resposta sem ficar perdendo muito tempo E sem enrolação tá aí aproveitem https://youtu.be/6C6fSe004R4
  • y = Cn,4/ Cn-1,3

    Combinação simples:

    y = n! / 4! (n-4)! * 3! [ (n -1)-3! ] / (n-1)! -> divisão de fração. Multiplicamos o primeiro pelo inverso do segundo

    y = n! / 4! (n-4)! * 3! (n-4)! / (n-1)!

    y = n*(n-1)! / 4 * 3! (n-4)! * 3! (n-4)! / (n-1)! -> depois de decompor, podemos cortar muitas expressôes

    y = n / 4

    Gabarito d


ID
5338186
Banca
Aeronáutica
Órgão
EEAR
Ano
2021
Provas
Disciplina
Matemática
Assuntos

Se log 2 = 0,3 e log 3 = 0,5, então o valor de log 0,0072/log 5 é

Alternativas
Comentários
  • Gabarito (A)

    • Dados:

    log 2 = 0,3

    log 3 = 0,5

    Vamos resolver por partes.

    1°) log 0,0072 = log 72 / 10000

    Fatorando fica:

    Log (2³. 3²) / 10^4

    Log 2³ + Log 3² - Log 10^4

    3Log 2 + 2Log 3 - 4 Log 10

    Substituindo os valores:

    3 . 0,3 + 2 . 0,5 - 4 . 1 = -2,1

    2°) log 5 é o mesmo que 10/2. Portanto:

    Log 10/2

    Log 10 - Log 2

    1 - 0,3 = 0,7

    3°) Agora é só dividir e correr para o abraço!

    -2,1 / 0,7 = -3

    Bons estudos!

  • Log 0,0072/log 5 = log (72/10000) / log 10/2;  

    Fazendo a decomposição do número 72 por fatores primos, teremos: 72 = 2³ x 3²; 10000 = 10^4 

    (Log( 2³ x 3²) - 4 x log 10) / (log 10 – log 2) = (3 x log 2 + 2 x log 3 – 4) / 1 – log 2; 

    Sendo log 2 = 0,3 e log 3 = 0,5 conforme o enunciado, teremos: 

    (3 x 0,3 + 2 x 0,5 – 4) / 1 – 0,3 = (0,9 + 1 – 4) / 0,7 = -2,1/0,7 = -21/7 = -3 

    GABARITO: A 

     

    @simplificandoquestoescombizus (Jefferson Lima)

     

     

  • *ESQUEÇAM O LOG 5, LEMBREM DELE APENAS NO FINAL DO CÁLCULO.

    *LEMBRE-SE QUE LOG 5 = 1 - LOG 2

    *LOG 2=0,3 E LOG 3=0,5

    LOG 0,0072 = LOG 72/10000 , SENDO ASSIM :

    LOG 72 - LOG 10000 =

    Log (2³. 3²) - 4 . LOG 10 =

    3.LOG 2 + 2.LOG 3 - 4.1

    3.0,3 + 2.0,5 - 4 =

    0,9 + 1 - 4 =

    -2,1

    AGORA JÁ FEITA A PRIMEIRA PARTE, HORA DE LEMBRAR DO LOG 5

    -2,1/LOG 5 =

    -2,1/LOG (10/2) =

    -2,1/ 1-0,3

    -2,1/ 0,7

    -3


ID
5338189
Banca
Aeronáutica
Órgão
EEAR
Ano
2021
Provas
Disciplina
Matemática
Assuntos

Uma bola é lançada verticalmente para cima. Se sua altura h, em metros, em relação ao solo, t segundos após o lançamento, considerando t [0,4] , pode ser calculada por h = −t2 + 2t + 8, então a altura máxima atingida pela bola é _____ m.

Alternativas
Comentários
  • Yv: – Δ/4.a

  • errei essa questão na prova por afobação kkk, a altura será dada pelo vertice do eixo y, conforme nosso colega acima descreveu


ID
5338195
Banca
Aeronáutica
Órgão
EEAR
Ano
2021
Provas
Disciplina
Matemática
Assuntos

Seja r a reta determinada por A (3, 5) e B (6, −1). O ponto de abscissa 8 pertencente à r possui ordenada igual a

Alternativas
Comentários
  • mr = (y-y°)/ (x-x°)

    mr = 5+1/3-6

    mr = 6/-3

    mr = -2

    Y- Y° = mr * (X-X°)

    Y - 5 = -2 (X -3)

    Y - 5 = - 2X + 6

    como x = 8

    então Y-5 = -2(8) +6

    Y - 5 = -16 + 6

    Y - 5 = -10

    y = - 5

  • Olá, a equação da reta é dada por:

     

    Y = ax+ b

     

    Do primeiro ponto temos:

     

    5 = 3a + b

     

    Vamos isolar o b:

     

    b = 5 – 3a

     

    Vamos fazer a mesma coisa com o segundo ponto:

     

    - 1 = 6a + b

     

    b = - 1 – 6ª

     

    Vamos igualar os dois valores de b e efetuar a equação:

     

    - 3a + 5 = - 1 – 6a

     

    3a = - 6

     

    a = - 2

     

    Substituindo em b:

     

    b = 5 – 3.(-2)

     

    b = 11

     

    E equação da reta é:

     

    Y = - 2x + 11

     

    A abscissa x = 8, vai nos dar a ordenada y em:

     

    Y = - 2.8 + 11

     

    Y = - 5

     

    Abraço e bons estudos!

  • Podemos resolver pelo alinhamento de 3 pontos, usando o famoso Det (para a condição de alinhamento de três pontos o determinante dos pontos tende a 0) . Perceba-se que há as abscissas dos 3 pontos e as ordenadas de 2 pontos, faltando justamente o "y" que ele quer.

    Temos as coordenadas (3,5) ; (6,-1) ; (8,y) ; portando:

    Det = / 3 5 /

    / 6 -1 / = 0 => 15+3y=0 => y= -5

    / 8 y /

    / 3 5 /


ID
5338198
Banca
Aeronáutica
Órgão
EEAR
Ano
2021
Provas
Disciplina
Matemática
Assuntos

A base de uma pirâmide é uma das faces de um cubo de aresta a. Se o volume do cubo somado com o volume da pirâmide é 2a3 , a altura da pirâmide é ________ da aresta a.

Alternativas
Comentários
  • Gab: B

    Volume do cubo= a^3

    Volume da pirâmide= a^2.h/3

    soma os dois conforme enunciado igual a 2a^3

    a^3+a^2.h/3=2a^3

    passa o a^3 pro outro lado

    a^2.h/3=2a^3-a^3

    a^2.h/3=a^3

    agr passa o 3 multiplicando

    a^2.h=3a^3

    passa o a^2 dividindo

    h=3a^3/a^2

    h=3a

    Não sei se ficou claro ou se é o jeito certo de fazer, espero que ajude, qualquer erro avise!

    Obs: ^ significa elevado

  • O volume do cubo é a^3. O volume da pirâmide é 1/3*Ah, em A que é a área da base, e h a altura.

    A base da pirâmide é a face do cubo, então A = a^2.

    Assim, a^3 + 1/3 * a^2 * h = 2a^3.

    Ou seja, h = 3a.


ID
5338201
Banca
Aeronáutica
Órgão
EEAR
Ano
2021
Provas
Disciplina
Matemática
Assuntos

Dadas as retas r: 2x − 3y + 9 = 0, s: 8x − 12y + 7 = 0 e t: 3x + 2y − 1 = 0, pode-se afirmar, corretamente, que

Alternativas
Comentários
  • r) y= 2x/3+3

    s) y= 2x/3 + 7/12

    t) y= -3x/2 + 1/2

    sabendo que a lei da função do 1º é f(x)= ax + b,

    sendo a o coeficiente angular

    sendo b o coeficiente linear

    para que sejam paralelas, o coeficiente angular deve ser o mesmo ( a1=a2 )

    para que sejam coincidentes, o coeficiente angular e o linear devem ser os mesmos ( a1= a2 e b1=b2 )

    para que sejam perpendiculares, o coeficiente angular de um deve ser o oposto do inverso do outro (a1.a2=-1), se a1= 2 e a2= -1/2

    r e s são paralelas ( 2/3 = 2/3 )

    r e t são perpendiculares ( 2/3 e -3/2 )

    s e t são perpendiculares ( 2/3 e -3/2 )

    Gabarito C


ID
5338204
Banca
Aeronáutica
Órgão
EEAR
Ano
2021
Provas
Disciplina
Física
Assuntos

A descoberta do elétron e sua interação com campos elétricos e magnéticos, através dos experimentos com a ampola de Crockes, possibilitou a idealização do modelo atômico de

Alternativas
Comentários
  • thomson

  • Química da Espcex me safando nessas questões de moderna kkkkk


ID
5338210
Banca
Aeronáutica
Órgão
EEAR
Ano
2021
Provas
Disciplina
Física
Assuntos

Alguns motoristas utilizam uma regra prática para manter uma distância de segurança ao veículo que vai à frente em uma estrada. Se os dois veículos estiverem percorrendo a mesma trajetória retilínea e no mesmo sentido, utiliza-se o intervalo de tempo em que os veículos passam por um ponto de referência no solo. Essa regra é feita utilizando um ponto fixo à beira da estrada, uma placa de sinalização, por exemplo, quando o veículo imediatamente à frente passar pelo ponto conta-se dois segundos até o veículo onde está o observador atingir o mesmo ponto de referência. Garantindo assim, uma distância de segurança entre os veículos.
Considerando dois carros com velocidades constantes de módulos iguais a 99 km/h e aplicando-se a regra prática descrita acima, qual será, em metros, a distância de separação entre os veículos?

Alternativas
Comentários
  • S=So+vt

    S=0+99.1/1800(Converti 2 segundos pra hora)

    S=55 metros ou 0,55 km

    Caso haja algum equívoco, corrijam-me.

  • Usando: Vm=S/T

    V=99km/h > 27,5m/s

    T=2s

    S= 27,5.2= 55,0 m

  • Ambos estão a 27,5m/s, se estão mantendo uma distância de 2s, então estão em uma distância de 27,5x2, que resulta em 55m.

  • S= VM. T

    S = 99 km . 2s = 198

    S= 196 /3,6 = 55m/s

    2s tempo de pausa ao frear

  • D= V x T

    99km/h= 99/3,6= 27metros/segundos

    27m/s x 2 segundos= 55 metros.

  • Usando a formula S=So+v.t

    S=espaço final

    So=espaço inicial

    v=velocidade

    t=tempo

    1) transformar km/h em m/s já que o tempo foi dado em segundos:

    99 / 3,6 = 27,5 m/s

    2) S=0 + 27,5 . 2

    S=55 metros

    GABARITO C.

  • 99 + 99= 198/3,6= 55

    3,6 pois de km para m se divide por 3,6


ID
5338219
Banca
Aeronáutica
Órgão
EEAR
Ano
2021
Provas
Disciplina
Física
Assuntos

Um móvel, de dimensões desprezíveis, parte do repouso e seu movimento retilíneo é observado por um professor de Física. Os valores das posições (x) desse móvel em função dos respectivos instantes de tempo (t) estão registrados na tabela a seguir. 

                                                                      x(m)       t(s)
                                                                        0           0
                                                                        2           1
                                                                        8           2
                                                                       18          3
                                                                       32          4
                                                                       50          5
                                                                       72          6 

 Podemos afirmar, corretamente, que o móvel executa um movimento retilíneo ____________________________________ . 

Alternativas
Comentários
  • S=so+vot + at^2/2

    *So=0

    *Vo=0

    72=a.6^2/2

    72=a.18

    a= 4m/s^2

  • Essa é a tabela caso na questão acima ela esteja bugada.

    x(m)/ t(s)

    0 / 0

    2 / 1

    8 / 2

    18 / 3

    32 / 4

    50 / 5

    72 / 6 

  • pq não pode ser v=vo+a.t???????

ID
5338222
Banca
Aeronáutica
Órgão
EEAR
Ano
2021
Provas
Disciplina
Física
Assuntos

Sobre um trilho de um banco óptico é colocado perpendicularmente um objeto real que se aproxima de um espelho. A imagem observada é sempre virtual e direita e o tamanho da imagem aumenta conforme o objeto se aproxima do espelho. Mas, mesmo a imagem aumentando de tamanho, é sempre menor que o tamanho do objeto. Pode-se afirmar corretamente que o espelho utilizado

Alternativas
Comentários
  • Todo espelho convexo é virtual e direta.

    Todo espelho côncavo é real e invertida

  • A questão informa primeiramente que o objeto aproxima-se do espelho e que a sua imagem é sempre virtual e direita.

    Até aí ok!

    Aí ela informa que a imagem sempre aumenta! se a questão parasse aqui, a resposta seria letra D.

    Porém no enunciado diz: "Mas, mesmo a imagem aumentando de tamanho, é sempre menor que o tamanho do objeto." Nós sabemos que imagem virtual de espelho côncavo é sempre maior que o objeto.

    Então a alternativa correta é a letra B.

  • ESPELHO CONVEXO VIRTUAL, DIREITA E MENOR

ID
5338225
Banca
Aeronáutica
Órgão
EEAR
Ano
2021
Provas
Disciplina
Física
Assuntos

Os satélites artificiais em órbita da Terra são expostos a ciclos severos de temperatura, pois durante metade da órbita recebem os raios solares intensos e na outra metade não recebem a radiação solar. Portanto, os satélites estão a uma temperatura muito alta na primeira metade da órbita e muito baixa na segunda metade.
Para simular as condições em que ficarão em órbita e verificar o funcionamento dos satélites nessas condições, são realizados testes em câmaras térmicas que, em baixa pressão, os expõem a muitos ciclos de temperatura.
Um determinado satélite foi testado em vários ciclos de -90°C a +90°C.
Essa variação de temperatura corresponde a uma faixa de _____ °F.

Alternativas
Comentários
  • ΔF = 1,8.ΔC

    ΔF = 1,8 (90 -(-90))

    ΔF = 1,8.180

    ΔF = 324

    GABARITO: LETRA D

    MEU ↯ CANAL ↯ NO YOUTUBE COM VÁRIAS QUESTÕES RESOLVIDAS

    https://www.youtube.com/c/ConcurseirodeElite

  • ele quer saber a variação lembrem-se sempre que a variação tem a formula diferente da de temperatura específica

    ΔC/5=ΔF/9 dado essa formula é só fazer a conta e você achara a resposta

    180/5=ΔF/9 =324


ID
5338228
Banca
Aeronáutica
Órgão
EEAR
Ano
2021
Provas
Disciplina
Física
Assuntos

Durante a pandemia da COVID-19, passou-se a usar na entrada dos lugares públicos um termômetro digital óptico para verificar se a pessoa que vai entrar no local não está no estado febril. Esse termômetro não necessita estar em contato com a pele da pessoa examinada, pois o mesmo mede a radiação térmica do corpo da pessoa.
É costume apontar para a testa de quem será examinado, pois normalmente é uma área que está descoberta. Porém, vários vídeos circularam nas redes sociais dizendo que essa prática era perigosa, pois os raios “emitidos” pelo termômetro, segundo os vídeos, poderiam prejudicar os neurônios das pessoas examinadas. Isso não tem nenhum fundamento, pois o termômetro não emite, mas sim, mede a irradiação eletromagnética emitida pela pessoa, através de um sensor ajustado para a faixa de frequência, cujo valor é proporcional à temperatura.
No espectro das ondas eletromagnéticas essa faixa de funcionamento do sensor do termômetro é chamada de

Alternativas
Comentários
  • termômetro que mede a temperatura apontado para a testa não emite raios infravermelhos, o objeto, faz a medição ao captar a radiação infravermelha emitida pelo próprio corpo humano


ID
5338231
Banca
Aeronáutica
Órgão
EEAR
Ano
2021
Provas
Disciplina
Física
Assuntos

Para estudar determinados fenômenos associados à luz é necessário utilizar corretamente a natureza dual atribuída à luz, ou seja, em determinados fenômenos a luz se apresenta como onda e, em outros, apresenta-se como corpúsculo. Com relação ao texto anterior, assinale a alternativa correta.

Alternativas
Comentários
  • Item c. O efeito fotoelétrico descreve o fenômeno de fótons retirarem elétrons de um corpo, esse modelo só é possível baseado no caráter corpuscular da luz, ou seja, que a luz é uma partícula. Já a difração é um fenômeno comum em quaisquer ondas, portanto pode ser corretamente explicado a partir do modelo ondulatório. Esses modelos se complementam pela dualidade onda-partícula da luz.

    Inclusive foi a descoberta do efeito fotoelétrico (embasado na dualidade) que levou Einstein a ganhar seu Nobel de Física.

  • valeu lara


ID
5338234
Banca
Aeronáutica
Órgão
EEAR
Ano
2021
Provas
Disciplina
Física
Assuntos

Os satélites geoestacionários realizam MCU (Movimento Circular Uniforme) com órbitas que pertencem a um plano imaginário que corta a Terra na Linha do Equador e que apresenta o mesmo período de rotação da Terra. Isso é obtido quando o satélite está a uma altura de 36000 km em relação a um ponto na superfície e com uma determinada velocidade tangencial. Se esse ponto situado na superfície, na Linha do Equador, apresenta uma velocidade tangencial de módulo igual a 1600 km/h, o módulo da velocidade tangencial de um satélite geoestacionário será de ______ km/h.
Utilize π = 3 .

Alternativas
Comentários
  • Informação importante: Se o satélite é geoestacionário, seu período de rotação deve ser igual o do ponto.

    Velocidade do ponto = 2πr/t

    1600 = 2.3.r/24

    1600 = r/4

    r = 6400 km

    Velocidade do satélite = 2πr/t

    Vs = 2.3.(6400 + 36000) / 24

    Vs = 42400/4

    Vs = 10600 km/h

    GABARITO: LETRA B

    MEU CANAL NO YOUTUBE COM VÁRIAS QUESTÕES RESOLVIDAS

    https://www.youtube.com/c/ConcurseirodeElite


ID
5338237
Banca
Aeronáutica
Órgão
EEAR
Ano
2021
Provas
Disciplina
Física
Assuntos

Em 24/09/2019 Victor Vescovo, explorador marítimo, conseguiu, apesar da alta pressão, atingir a profundidade de 5500 m na Fossa Molloy. Em contrapartida, devido à baixa pressão atmosférica, a uma altitude de 18000 m os líquidos presentes no corpo humano entram em ebulição mesmo estando a temperatura corporal normal, ou seja, 37°C. Essa altitude é chamada de Limite de Armstrong. Assinale a alternativa que indica corretamente a expressão da variação de pressão (ΔP) em função da variação de posição (Δh), tanto para altas altitudes como para grandes profundidades, e o respectivo referencial (sentido positivo indicado pela seta) para posição (h).
Considere que μ (densidade) é constante dentro de um mesmo meio; g (módulo da gravidade) é sempre constante e ΔP é sempre proporcional a Δh.

Alternativas
Comentários
  • A altura vai aumentando e a pressão atmosférica diminuindo, logo h>0 e P<0

    Gabarito letra A

  • alguém


ID
5338240
Banca
Aeronáutica
Órgão
EEAR
Ano
2021
Provas
Disciplina
Física
Assuntos

A luneta astronômica é um instrumento óptico destinado à observação de objetos celestes a grandes distâncias. Este instrumento consta basicamente de duas lentes, não justapostas e associadas coaxialmente, a objetiva e a ocular. Como o saudoso Prof. Dr. Alberto Gaspar escreveu em seu livro, a palavra objetiva pode ser entendida como uma abreviação da expressão “lente voltada para o objeto” e a palavra ocular está relacionada aos olhos. Sabe-se que a objetiva apresenta grande distância focal e a imagem conjugada é invertida e serve de objeto para a ocular. A imagem conjugada pela ocular é invertida com relação ao objeto celeste e maior com relação a imagem conjugada pela objetiva.
Portanto, pode-se concluir que:

Alternativas
Comentários
  • A questão diz que a lente objetiva inverte a imagem, portanto é convergente, uma vez que as lentes divergentes não fazem isso.

    A questão também informa que a lente ocular aumenta a imagem conjugada pela lente objetiva sem inverter a mesma, desse modo, entende-se que é convergente também, já que lente divergente não amplia a imagem, pelo contrário.

    Me corrijam se eu estiver errado, por favor!

  • Lente divergente só conjuga imagem virtual, direita e menor!!!!
  • Se as duas lentes possuem a imagem invertida em relação ao objeto, só podem ser convergentes, lentes divergentes não aceitam inversão, são somente menor, direita e virtual, GAB LETRA (B)


ID
5338246
Banca
Aeronáutica
Órgão
EEAR
Ano
2021
Provas
Disciplina
Física
Assuntos

Durante o desembarque das tropas aliadas nas praias da Normandia no Dia D, balões padronizados eram preenchidos com gás hidrogênio e presos por cabos à superfície para proteger as tropas contra ataques dos caças inimigos. Entre os gases disponíveis, o gás hidrogênio possui a menor densidade e é igual a 0,09 kg/m³, enquanto que o ar atmosférico possui densidade igual a 1,2 kg/m³. Considere que o módulo da aceleração da gravidade é constante com a altitude, e a densidade do balão é definida exclusivamente pelo gás que o preenche. Entre os gases disponíveis, para um mesmo volume preenchido do balão e sem o cabo estar sob ação da força de tração, ________________________, utilizando gás hidrogênio.
Assinale a alternativa que preenche corretamente a lacuna do texto anterior.

Alternativas
Comentários
  • Se o balão está preenchido com hidrogênio, que é muito menos denso que o ar atmosférico, e o cabo não está tensionado, haverá uma força resultante (empuxo) apontando pra cima.

    GABARITO: LETRA C

  • Pq não pode ser a B?

  • Por que a letra B está errada?


ID
5338249
Banca
Aeronáutica
Órgão
EEAR
Ano
2021
Provas
Disciplina
Física
Assuntos

A potência irradiada por uma lâmpada é distribuída em uma superfície esférica centralizada na lâmpada. A razão entre as potências por área, respectivamente, a 1 m e a 2 m da lâmpada é igual a

Alternativas
Comentários
  • 1º Calcular área dos círculos

    Círculo 1 - R= 1m

    Círculo 2 - R= 2m

    ÁREA=4pi.R^2

    A1=4pi.1^2

    A1=4pi

    A2=4pi.2^2

    A2=16pi

    2º Calcular a razão P/A

    R1= P/4pi

    R2=P/16pi

    3º Calcular R1/R2

    (P/4pi) / (P/16pi) = 4

    Letra B


ID
5338252
Banca
Aeronáutica
Órgão
EEAR
Ano
2021
Provas
Disciplina
Física
Assuntos

Duas amostras “A” e “B” de água no estado líquido de mesma massa (m) e mesmo calor específico (c) possuem temperatura iniciais diferentes TIA e TIB, sendo TIA maior que TIB. A mistura obtida com as duas amostras, após algum tempo, atinge a temperatura final TF. A quantidade de calor que a amostra “A” cedeu é igual a ____ .

Alternativas
Comentários
  • Primeiro nós temos que encontrar a temperatura final

    m.c.TiA + m.c.TiB = (m + m).c.Tf

    mc(TiA + TiB) = 2mcTf

    Tf = TiA + TiB/2

    Agora podemos calcular o que o enunciado pede

    Q = mc(TiA + TiB/2 - TiA)

    Q = mc(TiA + TiB - 2TiA/2)

    Q = mc(TiB - TiA/2)

    GABARITO: LETRA B

    Me acompanhe no YouTube, onde tenho diversas resoluções de questões, da EEAR, inclusive ↙

    https://www.youtube.com/c/ConcurseirodeElite


ID
5338255
Banca
Aeronáutica
Órgão
EEAR
Ano
2021
Provas
Disciplina
Física
Assuntos

Um garoto amarra uma pedra a um barbante e a faz girar em um plano vertical com uma rotação constante de 150 rpm (rotações por minuto). A sombra da pedra projetada no chão realiza um movimento de vai e vem em uma trajetória representada por um segmento de reta de 1,5 m de comprimento.
Considerando o movimento da sombra da pedra como um MHS com fase inicial nula, assinale a alternativa que apresenta corretamente a equação da elongação para esse movimento, no Sistema Internacional de Unidades.

Alternativas
Comentários
  • Equação da Posição: X(T) = A.cos(w.t + ω)

    A = A gente sabe quem ela é pq a questão diz que o movimento vai e vem é igual a uma reta de tamanho 1,5m, logo, cada pedaço vale 0,75m. Só pensar que é um balanço onde a ida + volta = 1,5m e eu quero só a ida ou só a volta.

    W = Deve transformar o 150 RPM em Frequência (Heartz), como o RPMinuto e a Frequência (Hz) é dada em segundos, só dividir por 60, resultando em 2,5 Hz. Após isso, usa-se a fórmula W = 2.π.F → W = 2.π.2,5 → W = 5π

    ω = "Considerando o movimento da sombra da pedra como um MHS com fase inicial nula", logo ω = 0

    X(T) = 0,75.cos(5πt) ALTERNATIVA B